Ophthalmology Fact Fixer 240MCQs with explanatoryanswers

Chung Nen Chua Li Wern Voon and Siddhartha Goel

Radcliffe Medical Press Radcliffe Medical Press Ltd 18Marcham Road Abingdon Oxon OX14 1AA United Kingdom www.radcliffe-oxford.com The Radcliffe Medical Press electronic catalogue and online ordering facility. Direct sales to anywhere in the world.

© 2002The authors

All rights reserved. No part of this publication may be reproduced, stored in a retrieval system or transmitted, in anyform or byany means, electronic, mechanical, photocopying, recording or otherwise without the prior permission of the copyright owner.

British Library Cataloguing in Publication Data

A catalogue record for this book is available from The British Library.

ISBN 1 85775908 7

Typesetby Aarontype Ltd, Easton, Bristol Printed and bound byTJ International Ltd, Padstow, Cornwall Preface

This book is designed to help residents, ophthalmologists and optometrists prepare for the MRCOphth, MRCS, FRCS and final optometry examinations respectively. It also serves as a guide to help comprehensive ophthalmologists update and expand their knowledge of ophthalmology. It is a useful aid for continuing medical education. The content includes updates in recent scientific trials and studies, medical thera­ peutics and surgical procedures, covering the subspecialities of and external disease, refractive surgery, cataract, glaucoma, orbital disease and oculoplastics, surgi­ cal and medical retina, paediatric ophthalmology, neuro-ophthalmology, uveitis, ocular oncology and pathology. To our knowledge, many of the other previously published self-assessment books have been found to be deficient in these aspects. A Fact Finder is provided at the back of the book.

Chung Nen Chua September 2002 About the authors

Chung Nen Chua Specialist Registrar Oxford Hospital Oxford

Li Wern Voon Clinical Corneal Fellow Oxford Eye Hospital and NuffieldLaboratory of Ophthalmology University of Oxford Oxford and

Registrar The Eye Institute Department of Ophthalmology Tan Tock Seng Hospital Singapore

Siddhartha Goel Ophthalmologist Kent County Ophthalmic and Aural Hospital Maidstone How to master MCQs

Multiple choice questions (MCQs) are an effective way of assessing the candidate's knowledge and are an important part of the final MRCOphth/MRCS. Apart from possessing good knowledge of the subject, we suggest the following tips which we hope will help you excel in the examination.

Remember to answer enough questions

The questions featured in the MRCOphth/MRCS/FRCS examinations are similar. Each question has five parts to be answered. Answers may be True, False or Don't know. + 1 mark is given for a correct answer and -1 mark for an incorrect one. No marks are given or deducted for an unanswered question. This system of marking is called nega­ tive marking. The fear of making mistakes may lead to answering too few questions. If the pass mark is 50%, and only half of the questions have been answered with confidence, one may score poorly, as a significant number of answered questions may be wrong. It is recommended that candidates attempt two-thirds of the questions. While this implies making guesses for some of the questions, an intelligent guess based on some back­ ground knowledge will more often be right than wrong.

Tricks used by the examiners

1 Pay attention to the phrasing of the question. Phrases that contain the words 'never' or 'always' are usually false, whereas phrases with the word 'may be' are usually true. Good MCQs tend to avoid using these words as much as possible. Other examples are words such as 'typical' or 'rarely'. A sentence may be correct in essence, but not when these phrases are used. For example: Retinoblastoma:

is inherited in the majorityof cases

arises from receptor cells of the retina

is associated with deletion of chromosome 13

is typicallypresent with pseudo hypopyon

that involves the optic nerve has poor prognosis.

Question illustrates the delicacy ofcorrect phrasing. Pseudohypopyon is a known presentation of retinoblastoma, however, it is not typical.

2 Beware of statements that contain dual information.The firstpart of the statement may be correct, whereas the second part is incorrect. Mistakes can easilybe made ifthe question is read in a hurry. For example:

In malignant hypertension:

intracranial pressure is raised

a macular star is caused bythe breakdown of the tight junction of the retinal pigment epithelium

there is bilateral optic disc swelling with a relative afferentpupillary defect

may show a dark choroid due to choroidal ischaemia

phaeochromocytoma is the most common cause. In this example, question <,~:L< contains a correct statement in the first part of the question (i.e.malignant hypertension causes bilateraloptic disc swelling) but an incorrect statement in the last part (i.e.relative pupillary defect is not a feature of malignant hypertension).

3 Numerical figures in questions are tricky,as they can easilybe altered to trick the unwary. Itis advisable not to answer these questions unless there is great certainty. For example:

The following are true about primary open-angle glaucoma:

relatives of the affected have an increased incidence of steroid-induced glaucoma

the incidence of a first-degreerelative developing the same condition is 1%

dilatation of the pupil always increases the intraocular pressure

abnormality of the trabecular meshwork is often observed during

visual field loss can occur in the presence of normal cup:disc ratio.

When answering this question, ()ne may vaguely remember the value 1% and be pleased to see it in question .The true answer is in fact 10%. 4 Avoid reading too much into the question. For example:

Iritisis a feature of:

systemic lupus erythematosus

psoriatic arthropathy

ulcerative colitis

Behcet's disease

rheumatoid arthritis.

While rheumatoid arthritis does not cause IrItiS,some may reason that rheumatoid arthritis-induced scleritis or peripheral corneal melts can give rise to cells in the anterior chamber and hence iritis.

5 Some easy questions may be rendered unanswerable bythe use of uncommon vocabulary. For example:

Deuterano maly:

is caused byan abnormal gene in the X-chromosome

is caused byan abnormality in the green pigment

occurs in about 5% of the European population

is more common than protanomaly is associated with decreased visual acuity.

In the question above, deuteranomaly is another term for red-green colour blindness. Ifone is uncertain of the medical term, it may lead to difficulties in answering the question.

Noting the pearls above will be useful In helping candidates excel In any multiple choice question examination.

1 Vigabatrin: 3 Kayser-Fleischer's ring:

is used as the first-linemedication in is pathognomonic of Wilson's grand mal epilepsy disease

increases GABA concentration in the first appears in the superior and retina inferior Descemet's membrane

causes visual field defects in one• occurs in all patients with hepatic third of patients failure due to Wilson's disease

is contraindicated in patients with occurs in all patients with neurologic pre-existingvisual field defect manifestations of Wilson's disease

causes narrowing of the retinal disappears with desferrioxamine arterioles. treatment.

2 The following pharmacological tests 4 Ocular signs associated with are useful in differentiatinga patient abnormal dentition are seen in: with a fixed dilated pupil: congenital syphilis 4% cocaine test

Reiger's syndrome 0.01% pilocarpine

congenital rubella 1% pilocarpine

Down's syndrome 2.5%phenylephrine

pseudoxanthoma elasticum. 0.1% adrenaline. 1 3

Vigabatrin is used in combination Kayser-Fleischer's ring is not with other anti-epilepticsin the pathognomonic for Wilson's disease; control of epilepsy.It is not used as a it can occur in other conditions such first-linetreatment except in West's as primary biliarycirrhosis. In syndrome.It increases the patients with suspected Wilson's concentration of GAB A in the brain disease,gonioscopy may be required and the retina.It is associated with to detect earlycopper deposition at visual field defects (as many as the periphery of Descemet's one-third of patients)but the membrane in the superior and mechanism is unknown. It should be inferior limbus. It may be absent in avoided in patients with pre-existing patients with hepatic failure due to visual field defect. Wilson's disease but is always appearance in patients taking present in those with neurologic vigabatrin includes narrowing of the manifestations.It disappears with retinal arterioles,wrinkles on the D-penicillamine treatment. retina surface and optic atrophy in the presence of visual field defects.

4

2 Congenital syphiliscan give rise to Hutchinson's teeth and Reiger's Cocaine test is used to confirm syndrome is associated with Horner's syndrome and 2.5% under-developed dentition. phenylephrine or 0.1% adrenaline are used to localise the site in Horner's syndrome.0.01 % pilocarpine causes constriction of Adie's pupil and 1% pilocarpine will constrict a dilated pupil caused by third nerve palsybut will not constrict a pharmacologically dilated pupil.

5 7

Internuclear ophthalmoplegia is Pigment dispersion syndrome is caused bya lesion in the ipsilateral associated with pigmentary medial longitudinal fasciculus, i.e.the glaucoma and the risk factors are same side as the eyethat has myopia and raised intraocular adduction restriction. Down-gaze pressure. In this condition, the palsyis caused bya lesion in the pigment dispersion is believed to be rostral interstitialnucleus of the caused bythe rubbing of the iris medial longitudinal fasciculus. The against the lens.This is caused by lesion for skew deviation is in the concavity of the iris as a result of a brain stem but the exact location is more posteriorlyinserted iris. unknown. Iridotomyis useful in reducing the iris-lenscontact and hence the amount of pigment dispersed.Lattice degeneration is increased in pigment dispersion syndrome as most of the

6 ••.•. ·.·.•.t.:.· .•.··.·.· .•.r..• sufferers are myopic. • <,,:fj::<'!i

IPev was firstdescribed in people of African descent but it is now being recognised in other races due to increased awareness. The condition 8 @)...•..·..a •.· ...· ...·.·· ..,'--· ..•.· .•• ~·.·,'---_.,<...... :b ...·• ..· .. ..· ..; .. ·.·.• ... · ~•.•...•...."_;",:Ow'o~ ...... •..•....•.; ....~...... ;. typicallyaffects women in the fifth and sixth decade. Haemorrhage is Aberrant regeneration of the common and it was previously called oculomotor nerve occurs in a posterior uveal bleeding syndrome. 'surgical' third nerve palsyand not in It is one of the differentialdiagnoses 'medical' third nerve palsysuch as of age-related macular degeneration diabetes mellitus or hypertension. (ARM D).Unlike ARMD, it is not Retraction of the upper lid on associated with drusen. The lesions down-gaze is called pseudo von are made up of dilated choroidal Graefe's sign and miosis of the pupil vessels and are best visualised with on adduction is termed pseudo indocyanine green. IPev tends to Argyll-Robertson pupil. affect the peripapillaryregion away from the fovea.Therefore, the outcome of treatment is more favourable than ARMD. 9 True autofluorescence is a feature of: 11 The following are true about the differences between LASIK (laser optic disc drusen assisted in-situ ) and PRK (photorefractive keratectomy):

macular drusen LASIK is technically easier to perform than PRK

cotton wool spots LASIK can be used to treat a higher myopia than PRK

myelinated nerve fibres LASIK causes less pain than PRK

astrocytic hamartoma. LASIK is associated with less regression than PRK

epithelial downgrowth is seen in LASIK but not PRK.

10 With regard to optic neuritis:

95% of patients recover their vision to 6/12or better 12 In Stickler's syndrome:

the risk of developing multiple typeII collagen is defective sclerosis is higher in patients with a T2 weighted abnormal MRI scan high myopia of more than 8diopters a faster visual recovery occurs with is a feature systemic corticosteroids bone-spicule pigmentary changes finalvisual outcome is improved with are common in the peripheral retina systemic corticosteroids retinal detachment affects about the risk of developing multiple 50% of patients sclerosis is reduced in patients treated with corticosteroids. epiphyseal dysplasiaof the long bones is a feature. 9 11

Autofluorescence is defined as the LASIK involves the creation of a emission of fluorescent light from partial thickness corneal flap ocular structures in the absence of followed byphotorefractive surgery. sodium fluorescein. On the other It is technically more difficult than hand, pseudo-autofluorescence PRK. However, it has the advantages results from reflection of light from offaster visual rehabilitation with less light-coloured or white fundal pain and the abilityto treat higher structures such as myelinated nerve myopia.Epithelial downgrowth can fibres,sclera, hard exudates or occur under the flap in LASIK, cotton wool spots. resulting in poor vision.

10 12

Systemic corticosteroids can speed Stickler's syndrome is an autosomal up the visual recovery of optic dominant condition. The patient has neuritis but the final visual outcome abnormal typeII collagen, with high does not appear to be affected. myopia (>8D),perivascular Itdoes not prevent the development pigmentary changes but no bone• of multiple sclerosis. spicule formation.There is empty vitreous and the incidence of retinal detachment is as high as 50%. Orofacial anomalies are common, as are jointchanges, especiallyat the epiphyses of the long bones. 13 In a patient with symptomatic right 15 The following are true about the superior oblique palsy,the following structures seen on gonioscopy: surgery may be useful: Sampaolesi's line is anterior to right inferior oblique anterior Schwalbe's line transposition Scheie's stripe is located within the leftinferior oblique myectomy trabecular meshwork

the scleral spur is the posterior leftinferior rectus recession border of the trabecular meshwork

Schwalbe's line marks the right superior rectus resection termination of Descemet's membrane

right superior oblique tuck. the scleral spur defines the anatomical limbus.

14 Increased pigmentary deposition in 16 Regarding Wernicke's the angles is seen in: encephalopathy:

pseudoexfoliation syndrome vitamin B1 deficiency is the underlying cause

patients on latanoprost treatment occurs in hyperemesis gravidarum

pigment dispersion syndrome lesions typicallyoccur in the frontal lobe

laser iridotomy ophthalmoplegia occurs as the result of peripheral cranial nerve palsies

oculodermal melanocytosis. cerebellar signs are characteristic. 13 15

A right superior oblique palsycan Sampaolesi's line is caused by result in vertical or torsional diplopia. pigment deposition and is found Vertical diplopiamay be treated with anterior to Schwalbe's line.Scheie's either ipsilateralinferior oblique stripe is outside the trabecular myectomy or anterior transposition, meshwork and refers to the area of contralateral inferior rectus recession pigmentation between the lens or ipsilateralsuperior oblique tuck zonules and the lens capsule. (sometimes in combination). The anterior border of the trabecular Torsional diplopia can be treated meshwork is defined bySchwalbe's with anterior transposition of the line and the scleral spur defines the superior oblique muscle. posterior border. The anatomical limbus is defined bySchwalbe's line.

14 16 Latanoprost increases melanin production in the melanocytes but Wernicke's encephalopathy is does not cause increased pigmentary characterised byophthalmoplegia, deposition in the angle. mental confusion and gait ataxia.It is caused byvitamin B1 deficiency and is commonly seen in alcoholics. It also occurs in hyperemesis gravidarum in which the pregnant woman cannot take in food due to recurrent vomiting. The ophthalmoplegia is central in origin. Lesions are found in the motor and vestibular nuclei, paraventricular region of the thalamus, hypothalamus and the cerebellum. 11 The following are true about Leber's 19 The following blood tests are useful hereditary optic neuropathy: for the respective diseases mentioned: it causes painless visual loss in young patients C-ANCA in Wegener's granulomatosis Uhthoff's phenomenon is a recognised symptom P-ANCA in polyarteritisnodosa

affected males do not pass the condition to their offspring anticentromere antibodyin scleroderma fluorescein angiography shows leakage of dyeat the optic disc anti-SS-B in primary Sjogren's syndrome the amount of recovery can be predicted from the genetic mutation. anti-striatedmuscle antibodyin thymoma.

18 The following signs are regarded as significant macular oedema in The Gram stain of a patient's corneal diabetes mellitus: ulcer reveals Gram-negative rods. The following are possible diagnoses: microaneurysms within one disc diameter of the fovea Pseudomonas aeruginosa

hard exudate in the fovea Corynebacterium

retinal thickening within 500p,m of the fovea Serratia marcescens

retinal thickening of one disc diameter, anypart of which is within Moraxella one disc diameter of the fovea

flamed haemorrhages within 500p,m Neisseria. of the fovea. 11 19

Painless visual loss differentiates ANCA is present in polyarteritis Leber's optic neuropathy from optic nodosa and Wegener's neuritis.Uhthoff's phenomenon granulomatosis. Perinuclear ANCA describes decreased vision with (P-ANCA) is seen in polyarteritis exercise or warming of body nodosa whereas cytoplasmic ANCA temperature and is seen in both optic (C-ANCA) is seen in Wegener's neuritis and Leber's hereditary optic granulomatosis. neuropathy, as well as in other optic neuropathies. The condition is caused byabnormalities in the mitochondrial DNA and therefore is not transmitted in sperm. Ophthalmoscopy reveals Corynebacterium is a Gram-positive circumpapillary telangiectatic rod and Neisseriais a Gram-negative microangiopathy and swelling of the coccus. Pseudomonas aeruginosa is nerve fibre layeraround the disc. the most common pathogen in However, unlike in patients with contact lens-related corneal ulcer. swollen disc, leakage of dyeis not Serratia marcescens is uncommon seen in fluorescein angiography.The but can cause rapid and extensive typeof mutation can predict visual corneal necrosis. Moraxellaspecies recovery, those with 117788mutation are opportunistic pathogens and are have poor recovery, whereas those usually seen in alcoholic or with 14484mutation have significant debilitatedpatients. visual recovery.

18

Clinicallysignificant macular oedema is defined as:

• retinal thickening within 500fJ,m of the fovea • presence of hard exudate with retinal thickening within 500fJ,m of the fovea • retinal thickening of one disc diameter and anypart of which is within one disc diameter of the fovea. In Miller-Fisher's syndrome: In infantileesotropia: two-thirds of patients present with amblyopiaoccurs in 40% of patients diplopia despite earlysurgery the ratio of male to female earlysurgical correction restores involvement is about 2:1 normal binocular single vision the cerebrospinal fluid shows asymmetric optokinetic lymphocytosis occurs with a better nasal-to-temporal movement than antibodyto ganglioside CQ 1 b is temporal-to-nasal movement diagnostic 90% of patients have dissociated chronic cases occur in 50% of vertical deviation patients. post-surgical accommodative esotropia is common.

With regard to progressive supranuclear palsy: The following are true about it occurs in patients with chronic retinoblastoma: Parkinson's disease a familyhistory is present in less than the vertical eye movements are 10% of the sufferers affected before the horizontal eye movements about 40% of all cases of retinoblastoma are heritable the downward pursuit movement is usually affected before the upward deletion of 13q14chromosome is pursuit movement seen in 25% of patients blinking to bright light is reduced systemic malformations are more common in those with bilateral than unilateral retinoblastoma oral levodopa improves the eye movement in the majorityof secondary cancers are more patients. common in non-heritable than heritable retinoblastoma. 21 23

Miller-Fisher's syndrome is a variant Infantileesotropia has large angle of Guillain Barre's syndrome.It is esotropia.Binocular single vision is characterised byareflexia, external usually poor despite earlytreatment. ophthalmoplegia and ataxia.Diplopia The temporal-to-nasal movement is is the most common presentation. usually better than nasal-to-temporal The cerebrospinal fluid typically movement on optokinetic testing. shows dissociation of proteins and Dissociated vertical deviation and cells, i.e.the protein is increased but inferior oblique overaction are there are minimal cells. Chronic or common and have been reported in recurrent cases are uncommon. about 90% of patients.Post-surgical esotropia may be caused byan accommodative component.

Progressive supranuclear palsyis a 24 form of parkinsonism, which is differentfrom Parkinson's disease. About 40% of all retinoblastomas The vertical eyemovement is are caused bygerminal mutation affected earlybefore the horizontal and are therefore heritable. Deletion eye movement. Both the saccades of 13q14occurs in about 1% of and pursuit movements are cases and is associated with systemic abnormal. The blinking reflex is also malformation. Heritable affected.Unlike Parkinson's disease, retinoblastoma is associated with a it does not respond to levodopa. higher incidence of secondary cancers. In shaken babysyndrome: The following are true about the use of pneumatic retinopexy: the average age of the patient is about 2 years old it is contraindicated in patients with glaucoma retinal haemorrhages are essential for the diagnosis it is not suitable for patients with inferior retinal detachment retinal haemorrhages are useful in determining the time of trauma presence of anyproliferative vitreoretinopathy is a neurological deficits occur in about contraindication one-third of patients the retinal breaks should be within permanent visual loss results from one clock hour of each other retinal lesions. the success rate is higher in pseudophakic compared with phakic patients.

Risk factors for expulsive suprachoroidal haemorrhage during cataract extraction include: The advantages of continuous high myopia curvilinear capsulorrhexis over capsulotomy include: history of glaucoma easier prolapse of the lens nucleus into the anterior chamber shallow anterior chamber ensures 'in the bag' intraocular lens implantation atherosclerosis allows hydrodissection of the lens with minimal risk of extension oftear to the equator blue iris. reduced incidence of post-operative intraocular lens decentration

reduced incidence of posterior capsule thickening. 27

Retinal haemorrhages are commonly Pneumatic retinopexyinvolves the seen in shaken babysyndrome. use of air to flattenthe retina However, they are not necessary for following cryotherapy.It is suitable diagnosis and should not be used to onlyfor superior retinal detachment determine the time of trauma. and the retinal tears should be within Neurological deficits occur in about one clock hour of each other. As one-third of patients.Visual there is often an increase in impairment is usually caused by intraocular pressure, it is cerebral damage. contraindicated in glaucoma. Grade C PVR is a contraindication as the retina is usually rigid in these cases. The success rate is higher in phakic than pseudophakic patients.

Expulsive suprachoroidal haemorrhage. is rare and has been estimated to occur in about one in 28 every thousand cases of cataract extraction. Risk factors include Capsulorrhexis allows safe increased axial length, glaucoma, hydrodissection and atherosclerosis and previous . It ensures expulsive suprachoroidal 'in-the-bag' intraocular lens haemorrhage. implantation and reduced post-operative decentration. Nucleus prolapse is difficult in capsulorrhexis compared with capsulotomy. Capsulorrhexis does not reduce the incidence ofposterior capsule thickening. In involutional ptosis: 31 The following lens changes are typical of the medical condition height of the skin crease is increased mentioned:

posterior subcapsular cataract in levator function is reduced retinitis pigmentosa

anterior subcapsular cataract in ptosis is decreased on down-gaze amiodarone user

posterior lenticonus in Alport's up-gaze is usually abnormal syndrome

droplet cataract in galactosaemia levator resection is the treatment of choice. polychromatic cataract in Wilson's disease.

An increased rate of proliferative vitreoretinopathy is seen in: In a patient with sub luxated lens,the multiple retinal breaks following tests are useful:

echocardiogram pneumatic retinopexy

serum methionine scleral buckling

brain MRI surgery

serum FT A antibodies vitreous haemorrhage.

urine thiosulphate. 31 ~...•....•....•.....•..•...... •.....•...... •..•...... • ~ In involutional ptosis,there is Amiodarone can give rise to anterior dehiscence of the levator stellate deposit but is usually visually aponeurosis resulting in an elevated insignificant.Alport's syndrome is skin crease. The ptosis is constant in associated with anterior lenticonus. up- or down-gaze.The up-gazeis not Polychromatic cataract is seen in affected as in some congenital ptosis. myotonic dystrophy. Levator advancement is the treatment of choice.

32

30 ••. .•.•.1.· •. Echocardiogram may show enlarged V:I!!JjJ~.··.~.·.·· !.•.· .• aortic roots or mitral valve prolapse Proliferative vitreoretinopathy is in Marfan's syndrome.High serum thought to be caused bythe release methionine is seen in of retinal pigment epithelium onto homocystinuria. Syphilis is a known the retina.The incidence of PVR is cause of ectopia lentis.Urine increased in large or multiple retinal thiosulphate is seen in sulphite breaks, cryotherapy,vitreous oxidase deficiency,in which the haemorrhages and vitrectomy disulphide bond is disrupted and can surgery.Pneumatic retinopexyand cause ectopia lentis. scleral buckling do not increase PVR. Shallowing of the anterior chamber The risk ofzonular dehiscence during during phacoemulsification may is increased in the indicate: following conditions: posterior capsule rupture pseudoexfoliation syndrome

retinal detachment pigment dispersion syndrome

suprachoroidal haemorrhage aniridia

inadequate infusion of BSS Ehler-Danlos syndrome

aqueous misdirection syndrome. albinism.

The following are true about the Concerning malignant glaucoma: Endophthalmitis Vitrectomy Study: it occurs most commonly after use of vancomycin in the irrigating filterationsurgery for acute solution reduces the risk of angle-closure glaucoma endophthalmitis the anterior chamber is typically Staphylococcusaureus is the most shallow common pathogen the volume of the vitreous is systemic corticosteroid is useful increased

the increased pressure is due to intravenous antibiotic improves the anterior displacement of the ciliary final visual outcome bodyby choroidal effusion vitrectomy is onlyuseful in patients the pressure can be controlled with a with light-perception vision. miotic. 35

Shallowing of the anterior chamber The zonules are weakened in can result from inadequate fluid patients with pseudoexfoliation infusion or a rise in vitreous pressure. syndrome, aniridia and Ehler-Danlos The latter can result from breath syndrome.Other conditions that can holding, pressure from the speculum, increase zonular dehiscence include or misdirection of the aqueous or high myopia,syphilis, Marfan's suprachoroidal haemorrhage. syndrome,Weil-Marchesani Posterior capsule rupture and retinal syndrome, sulphate oxidase detachment usually cause deepening deficiency and lysinedeficiency. of the anterior chamber.

36

~...•....~.•...... •.•...•.. : ~ Malignant glaucoma typicallyoccurs The study did not look into the use following in angle• of vancomycin or systemic closure glaucoma. The anterior corticosteroid in endophthalmitis. chamber is shallow with raised The most common pathogen is 5 intraocular pressure. The mechanism epidermidis. In the study, is believed to be caused byaqueous intravenous antibiotic did not appear misdirection, resulting in expansion to affect the outcome. Vitrectomy of the vitreous, causing forward appeared to improve the visual movement of the lens and the ciliary outcome in those with light body.The initialtreatment of choice perception only. is a cycloplegic which moves the lens backwards and encourages the flow of aqueous anteriorly. Uveitis and poliosis are seen in: The following are true about retinal capillary haemangioma: sympathetic ophthalmia it is usually the first manifestation of von Hippel-Lindau's syndrome Vogt-Koyanagi-Harada'ssyndrome radiological imaging of the abdomen and head should be carried out AZOOR irrespective of familyhistory

spontaneous regression is common tuberculosis and close observation is the management of choice sarcoidosis. there is usually a prominent feeding vessel but normal drainage vessel

a blood test may show thrombocythaemia.

With regard to acute retinal necrosis: the affected eyeis usually painless An HIV-positivepatient was found to have a white lesion on the retina. the condition is bilateral in one-third The following can accurately of cases differentiatea cotton wool spot from CMV retinitis: retinal detachment is an important cause of visual loss CD4+ count most patients are immunocompromised presence of vitritis gancyclovir is the treatment of choice. visual acuity

presence of haemorrhage

enlargement of the lesion. 37 39

Pigment abnormalities are seen in Retinal capillary haemangioma is sympathetic ophthalmia and VKH often the first manifestation of von syndrome,which can also cause Hippel-lindau's syndrome.As the uveitis. disease may skip a generation(s),a MRI scan of the abdomen and head for potentiallylethal tumours such as phaeochromocytoma or cerebellar haemangioblastoma is essential.The 38 lesions have variable progression, and ablation with either cryotherapy Acute retinal necrosis usually or photocoagulation is presents with a painful red eyeand recommended before the tumour pan uveitis. It usually affects gets too big.Both the feeding and immunocompetent patients.Herpes drainage vessels are usually enlarged. viruses, especiallyherpes simplex Cerebellar haemangioblastoma can and varicella zoster,are often give rise to polycythaemia through implicated. Visual loss is usually due erythropoietin secretion, and the to optic neuritis and retinal blood tests show increased red detachment. The treatment of blood cells. choice is initialintravenous acyclovir followed byan oral course. Gancyclovir is not used due to its toxicityand the need for intravenous 40 administration to be effective.

Early CMV retinitis can pose a diagnostic dilemma as it may resemble cotton wool spots which are common in HIV patients. Although CMV retinitis is more common in those with CD 4+ counts offewer than 50 cells per /11,it cannot be used to differentiatethe two conditions.Furthermore, earlyCMV retinitisis asymptomatic and vitritis is minimal. White lesions in HIV patients should be observed over time·for anyenlargement or development of haemorrhages which are suggestive of CMV retinitis. The following are true about the The following are true about optokinetic response: neonatal conjunctivitis: it tests the pursuit movement in the it is defined as conjunctivitis that direction of the movement of the occurs within 28days of birth drum chlamydial conjunctivitis is the most a subject who has a right pursuit common cause in the UK problem is likelyto have a lesion in the leftparietal lobe topical tetracycline is the treatment of choice in chlamydial conjunctivitis a subject who has a problem with of the neonate refixationwhen the drum is moved to the leftis likelyto have a lesion in Thayer-Martin medium is useful in the leftfrontal lobe isolating Neisseria gonorrhoea a patient with Parinaud's syndrome topical treatment should be will develop convergence retraction combined with systemic treatment nystagmus when the drum is rotated in gonococcal conjunctivitis. vertically in a patient with adduction deficit, the presence of adduction in response to an optokinetic drum In familial exudative indicates a supranuclear lesion. vitreoretinopathy:

males are more commonly affected Enlarged superior ophthalmic vein on than females CT scan is a feature in: dragged discs may be present which carotid cavernous fistula can be confused with retinopathy of prematurity posterior communicating artery fluorescein angiography shows failure aneurysm of vascularisation of the temporal retina craniopharyngioma exudative retinal detachment is the most common cause of poor vision von Hippel-Lindau's syndrome prophylactic cryotherapy of the peripheral retina is useful in cavernous sinus thrombosis. preventing visual loss. Optokinetic response tests the Chlamydial conjunctivitis is the most pursuit and saccadic movements. common cause of neonatal When the drum is rotated to the con junctivitis.Systemic antibiotics right, the pursuit movement is such as erythromycin are indicated mediated bythe right parietal lobe due to the high risk of pneumonitis. and the saccadic movement bythe Thayer-Martin medium contains right frontal lobe,i.e. the optokinetic antibiotic and antifungal agents that response tests the pursuit and will stop the growth of bacteria and saccadic movement on the side fungus but not that of Neisseria towards which the drum is moving. gonorrhoea. Like chlamydial Parinaud's syndrome causes up-gaze conjunctivitis, gonococcal palsyand vertical rotation of the conjunctivitis should also be treated drum will elicit convergence with systemic antibiotics. retraction nystagmus.In a patient with a frontal lobe lesion,there may be deviation of his or her eyesto the side of the lesion.However, provided the contralateral frontal lobe is 44 intact, rotating an optokinetic drum away from the side of the frontal Familial exudative vitreoretinopathy lobe will elicit a response. is an autosomal dominant condition. Clinicallyit may resemble retinopathy of prematurity with failure of vascularisation in the temporal retina leading to fibrovascular changes with dragged macula and disc. However, Enlarged superior ophthalmic vein a history of prematurity is absent. occurs when the drainage of venous Although exudates are common, blood into the cavernous sinus is retinal detachment is rare. Poor impaired. This occurs in carotid vision usually results from cavernous fistula and cavernous rheumatogenous retinal detachment. sinus thrombosis. The other The use of cryotherapy or conditions mentioned do not photocoagulation to the avascular increase the pressure of the retina reduces complications. cavernous sinus or impair venous drainage. 45 The following maculopathies 47 Presence of the following condition typicallygive rise to severe central decreases the risk of proliferative visual loss before the age of 40: diabetic retinopathy:

Sorby's macular dystrophy advanced glaucoma

Stargardt's macular dystrophy pseudoxanthoma elasticum

Best's disease high myopia

central areolar choroidal dystrophy posterior vitreous detachment

North Carolina macular dystrophy. renal failure.

46 Corneal blood staining following 48 The following are true about hyphaema is increased in: corticosteroid-induced ocular hypertension: raised intraocular pressure diabetes is a risk factor

a patient with sickle cell anaemia the ocular hypertension is caused by outflow obstruction prolonged hyphaema it is related to the strength of the topical steroid used Fuch's endothelial dystrophy the time from taking the steroid to development of ocular hypertension use of topical steroid. is longer for systemic than for topical steroid

ocular hypertension which failsto reverse after stopping corticosteroid is more common in myopia. 47

All the conditions mentioned are Proliferative diabetic retinopathy is dominantly inherited and give rise to the result of severe retinal ischaemia. severe central visual loss.Stargardt's Any condition that either reduces disease,North Carolina macular the number of nerve fibre layersor dystrophyand Best's disease usually increases oxygentransmission reduce the vision significantlybefore between the choroidal and retinal the age of 20.Sorby's macular circulation will reduce the dystrophycauses significant visual proliferation.Renal failure is loss in the 20s to 40s.Patients with associated with hypertension and central areolar choroidal dystrophy tends to exacerbate the retinal do not usually have significant visual ischaemia. loss until the 50s.

48 8.·...·.·...• ...• .•. •a 46 V@!JJ.••• •.· .. ••..•.· .. ·.· •..·.. t •.•.... Risk factors for corticosteroid• Corneal blood staining is a induced ocular hypertension include complication of hyphaema. Its glaucoma, relatives of glaucoma incidence is increased in patients patients,diabetes and myopia.The with prolonged hyphaema, large mechanism is outflow obstruction hyphaema, high intraocular pressure and may be caused bydeposition of and abnormal endothelium. Sickle mucopolysaccharide in the cell anaemia is associated with trabecular meshwork. It is related to delayedclearance of hyphaema due the potency of the steroid.Topical to sickling of the red blood cells and steroid causes ocular hypertension hence higher incidence of corneal earlier than systemic steroids.This is blood staining.The use of topical due to an increased ocular steroid does not increase corneal concentration after topical blood staining. administration compared with systemic administration.Stopping the steroid earlyin the course of the disease will reverse the ocular hypertension - however, irreversible ocular hypertension has been documented, especiallyin myopic patients. The following are true about thyroid With regard to congenital eyedisease: nasolacrimal duct obstruction: it is the commonest cause of about 6% of newborns have retraction symptoms referrable to dacryostenosis it is more severe in older patients the most common site of obstruction is at the valve of Hasner it is more severe in smokers compression of a dacryocystocele often produces a sticky discharge its severityis related to the degree of through the puncta thyrotoxicosis earlyprobing is recommended in the activity is best elucidated with patients with mucocele STER MRI sequence. the success rate of the initialprobing is about 60%.

The following are true about botulinum toxin: The following are true about giant the most commonly used botulinum cell arteritis: toxin in clinical practice is typeA visual loss is usually caused by it causes irreversible post-synaptic ischaemia of the posterior ciliary blockage at the neuromuscular artery junction visual loss is usually more severe the onset of action is about 72 hours than non-arteritic optic neuropathy

C-reactive protein is better than ESR prolonged use is associated with in monitoring the response to muscle atrophy treatment in a right abducence nerve palsy, during temporal arterybiopsy, the injection of botulinum into the right arterycan be found beneath the lateral rectus may reduce the temporalis fascia esotropia. the presence of skip lesion in the temporal arteryis associated with a less severe form of the disease. 51 (!&...... ~ .•...... •.•.•..••.••. ~ Thyroid eyedisease is more common It is estimated that congenital in females than males and tends nasolacrimal duct obstruction occurs to be more severe in the elderly in 73% of newborns but only6% are than the young.Although symptomatic.The most common site hyperthyroidism occurs in about of obstruction is at the valve of 80% of these patients,its severity Hasner. In a dacryocystocele,there is does not correlate with the severity concurrent obstruction in the of the eyedisease. STER MRI common canaliculus - therefore, sequence is useful in detecting the compression of the sac does not activity of the disease.The presence show reflux.A mucocele increases of 'wet' sequence suggests active the risk of orbital cellulitis and early disease. probing is recommended. The success of first probing in congenital nasolacrimal duct obstruction approaches 90%.

Botulinum toxin causes irreversible blockage of the pre-synaptic 52 neurone and thus inhibits the release of acetylcholine. The onset of action Branches of posterior ciliary artery is 72 hours, with a mean duration of supply the optic nerve head. action of 4 months. Termination of Ischaemia causes optic nerve head action is the result of regeneration of swelling. Compared with the new dendrites.Muscle atrophy non-arteritic form, the visual loss in occurs in prolonged use of giant cell arteritis is usually profound botulinum toxin.In right abducence and more often bilateral,unless nerve palsy,it can be injected into steroid treatment is carried out early. the non-paralysedmuscle, i.e.the C-reactive protein is a better right medial rectus, to reduce indicator of response to treatment contracture of the muscle and the because its concentration changes esotropia. more quickly with the amount of inflammation.The temporal artery lies on the temporalis fascia.Skip lesion may give a negative temporal arterybiopsy but does not correlate with the severityof the disease. In argon laser trabeculoplasty: In an eyewith raised intraocular pressure caused bySturge-Weber's laser is applied to the anterior half of syndrome,the pressure can be the trabecular meshwork effectivelylowered with: the amount of energy used per unit an a blocker such as brimonidine area of burn is larger than in retinal photocoagulation a f3 blocker such as timolol the amount of intraocular pressure reduction is related to the pre• treatment intraocular pressure latanoprost reduction of pressure is more marked in phakic than pseudophakic pilocarpine patients the effect on intraocular pressure topical acetazolamide. reduction becomes less with retreatment.

Laser iridotomy may be expected to The following are true about reduce the intraocular pressure in: nasolacrimal duct obstruction in the adult: phacomorphic glaucoma it is more common in females than in males phacolytic glaucoma presence of dacryoliths occurs in 25% of patients microspherophakia with glaucoma probing and syringinghas a success rate of 70% iris bombe in severe iritis external OCR has a higher success rate than endonasal OCR in relieving malignant glaucoma following epiphora cataract surgery.

OCR involves removing most of the ethmoid bone. 53 55

Argon laser trabeculoplasty increases Raised intraocular pressure in the outflow facilityof aqueous. It is Sturge-Weber's syndrome is caused applied to the anterior half of the byan increase in episcleral pressure trabecular meshwork. Application of that impairs aqueous outflow. a and laser to the posterior half risks the (3 blockers, as well as acetazolamide, development of anterior synechiae. suppress aqueous production and The effect on pressure reduction are therefore effective.Latanoprost is depends on the pre-treatment also effective as it increases the pressure, age of the patient and type uveoscleral outflow. Pilocarpine of glaucoma. It is less effective in increases the aqueous outflow pseudophakic and aphakic patients. through the trabecular meshwork The effect of pressure reduction and therefore is ineffective in the tends to decrease with time and presence of raised episcleral retreatment tends to be less pressure. effective.

56 54 Laser iridotomy is used in angle-closure glaucoma caused by Adult epiphora caused by pupillaryblock byproviding an nasolacrimal duct obstruction is alternative pathway for the aqueous. more common in females than Phacomorphic glaucoma is a males. It usually occurs in post• common contributing factor in menopausal women and there is primary angle closure. Pupillary block stenosis of the nasolacrimal duct. can be caused bythe abnormal Dacryoliths are uncommon. Unlike spherical lens in microspherophakia. congenital nasolacrimal duct Iris bombe impair the flow of obstruction, syringingand probing aqueous through the pupil and has a low success rate.External DCR hence pupillary block. gives a higher success rate than Phacolytic glaucoma is caused by endonasal DCR. During the the release of lens proteins into the operation, most of the bone drainage angle where they are removed to create a rhinostomy is engulfed bymacrophages, leading to from the lacrimal bone. blockage of the trabecular meshwork. Malignant glaucoma is caused byaqueous misdirection into the vitreous cavity. The following are true about Tenon's The following are true about cyst: exotropia: it is caused byadhesion between the intermittent exotropia improves with episclera and the Tenon's capsule time in the majorityof cases it usually appears as a thick-walled intermittent exotropia usually lobulated mass over the present with diplopia trabeculectomy site the angle of deviation tends to be digital pressure on the lower is larger for distant fixationcompared useful in breaking the cyst with near fixation injection of 5-fluorouracil is useful bilateral lateral rectus recession is more effective than resect/recess surgery in basic exotropia needling is contraindicated due to the risk of developing a persistent overcorrection of up to 10 prism non-healing fistula. dioptres prevents recurrence of exotropia.

The following steps can reduce the With regard to cataract extraction in effect of oculocardiac reflex during eyeswith uveitis: operation: prophylactic systemic steroid should peri bulbar anaesthesia be given to all patients with a history of uveitis to avoid post-operative macular oedema use of suxamethonium posterior capsule opacification is more common compared with the use of atropine general population

acrylic implants reduce the risk of increasing the blood oxygen giant cell formation on the lens saturation intraocular lens implant should be using light sedation. avoided in juvenile idiopathic arthritis

a large capsulorrhexis is recommended. due to the extra effortneeded to hold the two eyestogether or due to Tenon's cyst results from adhesion awareness of the strabismus. between the episclera and Tenon's Diplopia is not usually noticed because of suppression. Intermittent capsule, causing trapping of exotropia tends to have a larger the aqueous. It appears as a dome-shaped mass over the angle of deviation at distant compared with near fixation(basic operated site.Digital pressure either exotropia). basic exotropia, over it or on the lower globe may In break the adhesion. Ifthis fails, resect/recess surgery is the treatment of choice whereas in needling may be used to break the simulated exotropia,bilateral lateral adhesion. Revision of the bleb or rectus recession appears to be more repeat trabeculectomy with antimetabolite agents may be effective.Initial overcorrection of up to 10 prism dioptres reduces the required. incidence of recurrence. The initial esotropia tends to lessen with time.

Oculocardiac reflex occurs during manipulation of the eye,especially the extraocular muscle during Prophylactic systemic steroid strabismus or retinal surgery.Both treatment is onlyrecommended in the heart rate and blood pressure patients with previous macular drop when this happens. The use of oedema caused bythe uveitis. peribulbar anaesthesia reduces the Post-operative uveitis and posterior transmission of nerve impulses from capsule opacification are more the eyeand therefore dampens the common compared with the general reflex.The use of atropine prevents population.To prevent deposits of bradycardia and is used to abolish materials on the lens, either an the reflex.Other measures acrylic- or heparin-coated lens may mentioned are ineffective. be used. Lens implant in juvenile idiopathic arthritis is associated with fibrovascular proliferationover the lens and risk of phthisis bulbi.A large capsulorrhexis reduces the incidence of synechiae between the iris and Intermittent exotropia seldom the lens capsule. resolves spontaneously.The majority ofpatients present with aesthenopia The following are true about the The following are true about development of orbital emphysema strabismus surgery: following orbital injury: during a resect/recess operation, it is most often seen in orbital floor resection should be performed fracture before recession prophylactic antibiotic is needed to operating on the medial rectus has a prevent orbital cellulitis more significant effect on the amount of strabismus correction CT scan can usually identifythe compared with the lateral rectus location a combined resect/recess operation infraorbitalnerve anaesthesia is a has a greater effect than if the two feature procedures are carried out on separate occasions repair is needed in about 50% of cases. adjustable suture should be avoided in inferior rectus surgery

resection of the superior rectus is associated with lid retraction. The following are true about lattice degeneration: it is found in 8% of the general population The findings of the following are in favour of neurogenic rather than the vitreous over the lesion is firmly mechanical muscle palsy: attached to the retina duction of the affected eyeresults in atrophic holes within the lattice more movement than version degeneration are the most common cause of retinal detachment Hess chart of the affected eyeis smaller about 50% of patients with retinal detachment have lattice the intraocular pressure is normal in degeneration all directions of gaze sclerosed blood vessels occur within the absence of retraction of the the lattice degeneration. globe

abnormal head posture may be adopted to reduce diplopia. muscle insertion to a more posterior position.Recession is reversible, Orbital emphysema usually results whereas resection is not; therefore, from medial wall fractures such that recession should always be sneezingor nose-blowing forces air performed before resection in case from the paranasal sinuses into the of iatrogenic mishaps such as erroneous identification of the orbital tissue. Prophylactic antibiotic prevents orbital cellulitis.The muscle. The medial rectus is stronger fracture is often small and difficult to than the lateral rectus and, therefore, operation on this muscle has a locate even with a CT scan. The greater effect.The inferior rectus majorityof cases resolve spontaneously and surgery is rarely muscle has a tendency to slip into the orbit when detached - required. Infraorbitalnerve anaesthesia is a feature of orbital therefore, adjustable sutures are not recommended. Superior rectus floor fractures. resection can cause upper lid retraction.

Lattice degeneration occurs in 8% of the population but onlya small minority of these patients develop Duction refers to movement of one retinal detachment. However, it is eye,whereas version refers to found in 50% of patients with retinal conjugate movement of both . detachment. The cause of Because of Hering's law of equal detachment is usually due to a innervation, conjugate eye retinal tear surrounding the lattice movement of the affected eyeis less degeneration, where the vitreous is than duction. Hess charts in both firmlyattached to the retina.Over cases are reduced - however, in a the lesion,the vitreous is typically neurogenic palsy,there is liquefied.Pigmentation and sclerosed proportional spacing between the vessels are common features within inner and the outer fields.Intraocular the lesion. pressures are elevated in a mechanical muscle palsywhen the patient looks away from the site of the mechanical lesion.Globe 63 retraction occurs in a mechanical palsywhen the patient looks away Resection involves cutting and from the site of the lesion.Abnormal shortening of the muscle, whereas head posture may be adopted in both recession involves moving the conditions. The following are true about the severityof diabetic retinopathy is myelinated nerve fibres in the eye: related to the control of diabetes myelinationof the optic tract begins mellitus at the optic nerve head before birth the probabilityof progression from myelination is produced by pre-proliferativeto proliferative astrocytes diabetic retinopathy is 50% over a 2-yearperiod it is more common in males than females the major cause of visual loss in diabetic retinopathy is vitreous it has no visual consequence haemorrhage

aspirin is useful in delayingthe spontaneous disappearance occurs progression of diabetic retinopathy. in multiple sclerosis.

In retinal detachment caused by The following are true about retinal dialysis: retinopathy of prematurity: trauma is the major cause screening should be performed for all babies born with a weight of less the most common site is the than 1500gor less than 31 weeks of inferotemporal region gestational age folding of the posterior edge of the retinopathy of prematurity typically retinal dialysisis common occurs at about 4 weeks afterdelivery the prognosis is poorer than for the rush disease refers to the presence of patient with retinal detachment vitreous cells and tortuous blood caused byretinal tear vessels

high scleral buckling is essential for threshold disease is determined by successful closure of retinal dialysis. the stage as well as the zone of the disease

increased kappa angle is seen in The following are true about diabetic patients with cicatricial stage of the retinopathy: disease. diabetic retinopathy is rare before the onset of puberty Myelination of the optic tract begins Diabetic retinopathy occurs in 50% at the lateral geniculate body and is of patients with diabetes mellitus produced by the oligodendrocytes. who have had the disease for more The incidence is about 0.3% and than 7 years. The Diabetic Control affects males more than females. It is and Complications Trial shows that associated with amblyopia and good diabetic control significantly myopia. In multiple sclerosis, reduces the incidence of diabetic demyelination occurs and may cause retinopathy. While the visual loss spontaneous resolution of the from vitreous haemorrhage is often condition. profound, the major cause of visual loss in diabetic retinopathy is macular oedema. The Early Treatment of Diabetic Retinopathy Study shows that aspirin has no effect on either non-proliferative or proliferative diabetic retinopathy. The most common cause of retinal dialysis is trauma. The most common site is the inferotemporal region

~""..'."'.'.... '..•'.'." •...'..' .. '•..' .•. '...•. " .....•.•..•..... followed by the superonasal region. ~ The posterior edge of the retinal Risk factors for retinopathy of dialysis remains attached to the prematurity (ROP) are low birth vitreous base, which prevents it from weight «1500g) and early gestational curling over, unlike a giant retinal age «31 weeks). ROP typically tear. Compared with retinal occurs 6 to 8 weeks after delivery and detachment, the prognosis for retinal is estimated to occur in about 5% of dialysis is good. High buckling should all premature babies. Rush disease be avoided during the operation as refers to rapid progression of ROP, this would cause fish-mouthing and whereas plus disease refers to the prevent closure. presence of vitreous cells and tortuous blood vessels. Threshold disease is defined as stage 3 ROP with 5 or more contiguous, or 8 or more interrupted, clock hours in the presence of plus disease in zone I or II. Increased kappa angle occurs in cicatricial stages of the disease, with dragged disc causing the fovea to be displaced laterally. The following features favour a grid laser photocoagulation is useful diagnosis of choroidal melanoma in the presence of macular oedema rather than naevus: with a visual acuity of 6/18. a height of more than 3mm on ultrasound high internal reflectivityon A-scan The following are true about the management of branch retinal vein occlusion: presence of drusen on the surface laser treatment should be carried out within 3 months from the onset of visual field defects the event to be effective

clinical diagnosis is usually sufficient presence of choroidal neovascular to decide on the advantage of laser membrane. treatment in a patient with macular oedema

patients with hypertension are The following are true about central unlikely to benefit from laser retinal vein occlusion: treatment haematological disorders are more ifthe vision is less than 6/60,macular common in patients less than 60 laser is unlikely to be beneficial yearsof age than those over 60 years laser treatment should be carried out of age in the presence of retinal non• the prognosis for younger patients is perfusion of 5 disc diameter or more , better than for older patients based on the fluorescein angiography. the Central Retinal Vein Occlusion Study (CRVOS) shows that aspirin can prevent recurrence in the affected eyeor involvement of the fellow eye the Central Vein Occlusion Study shows clear benefit of prophylactic laser treatment in ischaemic eyes 69 photocoagulation in macular oedema with a visual acuity of The most important differential 6/18has not been found to be useful diagnosis of choroidal melanoma is in this study. the choroidal naevus. There are certain features that favour the diagnosis of choroidal melanoma and these include: 71 • a height of more than 3mm • overlyingorange pigment in the Based on the findings of the Branch retinal pigment epithelium Retinal Vein Occlusion Study Group, (lipofuscin)as compared to drusen laser treatment is useful in macular in choroidal naevi oedema secondary to branch retinal • presence of subretinal fluid vein occlusion if:first, the foveal • high internal reflectivityon A-scan vasculature is intact and, second, the • visual field defect. vision is between 6/12and 6/60. Choroidal neovascular membrane Fluorescein angiography is essential can occur in both conditions and are in identifyingpatients who would of littleuse in differentiation. benefitfrom laser treatment. Laser treatment should also be delayed until 3 to 6 months after the event to allow for spontaneous resolution of oedema and intraretinal 70 haemorrhages. While retinal non-perfusion of 5 disc diameter In central retinal vein occlusion, or more increases the risk of haematological disorders such as peripheral neovascularisation, laser protein S or C deficiency are more need not be performed provided the commonly detected in patients patients can be followed up at under the age of 60.Compared with regular intervals. the older age group, the visual prognosis for younger patients is better.The Central Retinal Vein Occlusion Study shows no distinct advantage of prophylactic laser treatment in ischaemic eyes provided the patients can be followed up regularly for the development of neovascularisation. The effect of aspirin was not considered in this study.Grid In the management of argon laser is more effective than cytomegalovirus (CMV) retinitis with diode laser in retinal pan• HAART (highly active anti-retroviral photocoagulation. therapy):

HAART compresses steroids and gancyclovir The following are true for the HAART improves the survival rate of phakomatoses and the location of AIDS patients with CMV retinitis their abnormal gene:

CMV virus is eradicated typeI neurofibromatosis - chromosome 21

the risk of opportunistic infection is tuberous sclerosis - chromosome 9 reduced

severe vitritis is a complication. von Hippel-Lindau's syndrome - chromosome 3

ataxic telangiectasia - chromosome 13

The following are the guidelines Wyburn-Mason syndrome - based on the Diabetic Retinopathy X chromosome. Study (DRS):

immediate laser treatment should be administered to eyeswith optic disc neovascularisation, irrespective of size

immediate laser treatment should be administered to all pregnant women with severe non-proliferativediabetic retinopathy

pan-retinal photocoagulation should be administered in one session for maximal effect

retinal neovascularisation should never be lasered directly which is at least one-half disc area in extent with pre-retinal or HAART involves the combination of vitreous haemorrhage. anti-retroviral drugs such as • Pan-photocoagulation should be nucleoside reverse transcriptase divided into several sessions in inhibitors (nucleosides),non• close succession rather than in a nucleoside reverse transcriptase single visit,to prevent inhibitors (NNRTI)and protease complications such as macular inhibitors (PI).It has been shown to oedema. improve the survival rate of AIDS • Direct laser can be applied to flat neovascularisation elsewhere but patients with CMV retinitis.During treatment, the CD4 + T lymphocytes not to disc vessels or vessels elsewhere which are elevated. are increased and the risk of opportunistic infection is reduced. In addition,the time interval between relapses of CMV retinitis is increased. However, the CMV virus is not There are nine conditions classified eradicated but merely suppressed. under phakomatoses and their One of the commonly encountered genetic loci are as follows: complications of HAART is severe • neurofibromatosis typeI - vitritis,thought to be caused bythe autosomal dominant - restored T lymphocytes which act chromosome 17 against the CMV virus. Vitritis can • typeII - autosomal dominant - lead to cystoid macular oedema, chromosome 22 which impairs the patient'svision. • tuberous sclerosis - autosomal dominant - chromosome 9 None • Sturge-Weber's syndrome - sporadic The guidelines based on the DRS are • von Hippel-Lindau's syndrome - as follows. autosomal dominant - • Immediate laser treatment should chromosome 3 be given to an eyewith optic disc • Louis-Bar syndrome-autosomal neovascularisation which is recessive - chromosome 11 associated with retinal or vitreous • Wyburn-Mason syndrome - haemorrhage, and in the absence sporadic ofthe latter when the extent ofdisc • Klippel-Trenaunay-Weber neovascularisation is at least syndrome - autosomal one-quarter to one-third disc area. dominant - chromosome • Laser treatment is also unknown recommended for • cutis marmarata telangiectasia neovascularisation elsewhere congenita - sporadic. 75 The following are true about 77 With regard to central serous systemic drugs used in the treatment retinopathy: of glaucoma: it is associated with the use of acetazolamide causes tachypnoea inhaled steroid

90% show classical smoke stack acetazolamide should be avoided in appearance on fluorescein patients with hepatic failure angiography

mannitol should be warmed to room optic disc pit is present in 25% of temperature before intravenous cases administration to avoid formation of crystals laser photocoagulation reduces the incidence of recurrence glycerol is administered byrapid intravenous infusion laser photocoagulation increases the risk of choroidal neovascularisation. glycerol is more effective than mannitol in lowering intraocular pressure.

78 In the following conditions, there is reduction of ERG b-wave amplitude as compared to the a-wave: 76 The following are true about macular holes: siderosis

epiretinal membrane is commonly associated with macular hole Best's disease

onlyhalf of stage 1 macular holes progress central retinal vein occlusion

Watzke-Allen's sign is positive in stage 1 macular hole vigabatrin toxicity

posterior vitreous detachment occurs in stage 3 macular hole cancer-associated retinopathy.

following successful macular hole surgery,cataract is a common cause of poor vision. 77

Acetazolamide causes metabolic Central serous retinopathy gives rise acidosis which in turn causes to sensoryretinal detachment due to tachypnoea. It should be avoided in an abnormality of the retinal patients with renal or hepatic failure pigment epithelium. The cause is as it can cause fatal acid-base unknown. It is more common in imbalance. Mannitol is an effective males than in females and is osmotic diuresis and more effective associated with a typeA personality. than glycerol in lowering the Systemic and inhaled steroids have intraocular pressure. Glycerol should also been implicated. While a smoke not be given intravenously as it stack is a classical picture in causes severe vasoconstriction ofthe fluorescein angiography of this afferentglomerular arterioles and condition, it is seen in only10% of resultant haematuria. cases. The rest give an ink blot appearance. Laser photocoagulation can speed up recovery but does not prevent recurrence. There is a small risk (2%)of inducing choroidal 76 neovascularisation with laser.

Epiretinal membrane can give rise to pseudo hole and is not a feature of idiopathic macular hole. About 50% of stage 1 macular holes progress. 78 Watzke-Allen's sign,in which a break in the slit beam is observed, occurs Reduction of b-wave with relative in full-thickness hole, usually at normal a-wave occurs in disorders stage 3 and 4.Posterior vitreous that result in a diffuse degeneration detachment occurs in stage 4. or dysfunction of cells in the Cataract is a common complication inner nuclear layer(Muller or of vitrectomy and a significant cause bipolar cells).Best's disease of poor vision following macular typicallyshows abnormal EOG and hole surgery. cancer-associated retinopathy causes generalised reduction of a- and b-waves. 79 The following are true about the 81 In pseudoexfoliation syndrome: visual evoked potential (VEP): deposition of pigment in the it is abnormal in patients with poor trabecular meshwork often precedes vision due to age-related macular the appearance of dandruff-like degeneration material on the lens

delayedP100 latency with normal pupil dilatation is usually poor with visual acuity is a feature ofrecovered mydriatics optic neuritis peripheral transillumination is a it is useful in detecting early common feature glaucomatous changes chronic open angle glaucoma occurs the use of miotics can affect the in about 50% of cases result raised intraocular pressure should ischaemic optic neuropathy causes a always be treated, irrespective of decrease in YEP amplitude in the visual field loss. presence of normal latency.

80 In albinism:

the number ofmelanocytes is usually normal

visual acuity is better in tyrosine• positive than tyrosine-negative oculocutaneous albinism

visual evoked potential is always abnormal

bleeding diathesis due to poor platelet aggregation is a feature of Hermansky-Pudlak syndrome

recurrent pyogenic infection is a feature in Chediak-Higashi syndrome. • Chediak-Higashi syndrome: recurrent pyogenic infection and Visual evoked potential (VEP) is a bleeding diathesis cortical response to pattern or flash • Hermansky-Pudlak syndrome: stimuli. The two parameters analysed bleeding diatheses, pulmonary are the latency and the amplitude. fibrosis,inflammatory bowel A delayedlatency of pattern VEP is disease,renal failure and suggestive of a demyelination cardiomyopathy. process. The amplitude of pattern The bleeding diathesis in Hermansky• VEP is useful, especiallyin children Pudlak syndrome is due to abnormal because it correlates well aggregation of platelets. with visual acuity.Patients with abnormal visual acuity such as age-related macular degeneration will have abnormal VEPs. VEPs are not useful for visual field defects.In miotic pupils, the amount of retinal illumination is reduced and this can Pseudoexfoliation syndrome is an give rise to delayedlatency. important cause of secondary Therefore, it is important to open-angle glaucoma. Although the standardise the testing conditions. appearance of dandruff-like material is characteristic of this condition, the earliest sign is the deposition of pigment in the trabecular meshwork, lens and iris.Pupil dilatation is poor and cataract extraction may be Albinism is a group of genetic difficult.Complications of cataract disorders with hypopigmentation of surgery are also increased due to the eye,skin and hair weakened zonules,which can lead (oculocutaneous type)or the eye to zonulysis andvitreous loss. only(ocular type).The In about 50% of patients,chronic oculocutaneous typeis inherited in open angle glaucoma occurs. an autosomal recessive fashion, and Intraocular pressures are often the ocular typein an X-linked unstable and can lead to severe fashion.Patients with pigment glaucoma. Therefore, anyevidence production, i.e.tyrosine-positive of raised intraocular pressure should type,tend to have better vision. be treated early. Visual evoked potential is abnormal and there is increased decussation of the optic nerve. Two typesof syndromes have been described for oculocutaneous albinism: In progressive outer retinal necrosis The following are true about (PORN): sympathetic ophthalmia: the disease is preceded by bilateralgranulomatous pan-uveitis is ophthalmic shingles in the majority a feature of patients fluorescein angiography is useful for this condition is found almost diagnostic purposes exclusively in AIDS patients removal of the injured eyeshould be involvement of the macula occurs performed within a week to avoid earlyin the course of the disease this complication vitritis is usually severe the choroid is typicallythickened with occlusion of the choriocapillaris intravenous acyclovir is usually Dalen-Fuchs' nodules are found on effective in controlling the disease. the retinal side of Bruch's membrane.

In a child who presents with leukocoria, the following features are in favour of retinoblastoma rather than toxocariasis: involvement of both eyes

presence of calcification

age of less than 2 years

presence of hypopyon

male sex. 82 84

PORN is caused bythe varicella Sympathetic ophthalmia occurs zostervirus, resulting in inflammation most commonly following a of the outer retina.Preceding perforating injurybut has been ophthalmic shingles is not common. described in ophthalmic procedures The disease has onlybeen described such as cataract surgery and in AIDS patients.Clinically, there is .Sympathetic earlyinvolvement ofthe macula with ophthalmia is a clinical diagnosis. littleor no vitritis.Antiviral Fluorescein angiography does not treatment, including acyclovir, has show features typical of this not been effective and severe visual condition. Although sympathetic loss is common. ophthalmia has been reported as earlyas 5 days,the general consensus is to remove an injured sightless eyewithin 2 weeks of trauma. Thickening of the choroid is 83 caused bycellular infiltrationbut the choriocapillaris are not involved. Retinoblastoma can be bilateral, Dalen-Fuchs' nodules contain whereas toxocariasis has not been aggregates of lymphocytes and reported to affect both eyes. macrophages. They are found on the Calcification of the lesion is typically retinal side ofBruch's membrane. Itis seen in retinoblastoma.Toxocariasis not pathognomonic of the condition is acquired from ingestion of ova and as it is also seen in Vogt-Koyanagi• tends to occur in older patients. Harada's syndrome. Hypopyonfrom severe inflammation can occur in both diseases.There is no gender predilection for either condition. With regard to acanthoamoeba In a patient with a pale optic disc, keratitis: the following test may be useful in reaching a diagnosis: it is equally common in rigid gas permeable contact lens users as in full blood count and film soft contact lens users the most common typesare A serum folate level hatchetti swimming with contact lenses is a colour vision test risk factor radial perineuritis is diagnostic serum FT A Abs

the cyst wall can be visualised with visual evoked potential. ultraviolet light following staining with calcofluor white.

In sebaceous cell carcinoma of the eyelid: unilateral blepharitis is a common feature previous eyelidradiation is a risk factor the upper lid is more commonly involved than the lower lid compared with non-ocular sebaceous cell carcinoma, those affectingthe eyelidstend to be more aggressive

Sudan black is used to differentiate the tumour from squamous cell carcinoma. involved than the lower lid due to the larger number of meibomian Acanthoamoeba keratitis is caused glands in the upper lids.Risk factors bya free-livingprotozoan. The most include old age and previous common species involved are radiation to the head. In a poorly differentiatedtumour, it may be A polyphagaand A castellani. Improper disinfection of the contact difficult to differentiateit from lenses is a major risk factor as is squamous cell carcinoma. However, the use of Sudan black or Oil red 0 swimming with contact lenses. can be useful as both will stain for Although both rigid gas permeable and soft contact lenses have been the presence of fat in tumour cells seen in sebaceous cell carcinoma. implicated in acanthoamoeba keratitis,the incidence is higher amongst soft contact lens users. Clinically,the condition is very painful for the amount of clinical 87 signs seen.Initially, it can present with dendrites which can be Optic atrophy is a physical sign and mistaken for herpes simplex keratitis. further tests are needed to find the Radial perineuritis is a characteristic underlying cause. Vitamin B12 and diagnostic feature.The deficiency is a cause of optic protozoan can be visualised under atrophy and a full blood count ultraviolet light after staining with may show macrocytosis and calcofluor white. In vivo visualisation hypersegmentation of the can be achieved with confocal neutrophils. Serum folate may give microscopy. The medium used the same haematological pictures to culture the organism is a but optic atrophy is not a feature nutrient-poor agar overlaid with of folatedeficiency. Positive serum E coli. FT A Abs may suggest syphilisas a cause of the optic atrophy. Colour vision test and visual evoked potential are abnormal in optic atrophy and do not provide information about the underlying cause. Sebaceous cell carcinoma of the upper lid is an aggressive tumour which can metastasise early.It typicallypresents as unilateral blepharitis or a recurrent chalazion. The upper lid is more commonly In essential blepharospasm: The following are true about mitomycin C: symptoms usually begin as excessive blinking it is isolated from a fungus symptoms usually improve during sleep it inhibits DNA synthesis the presence of facial grimace and retrocollic spasm constitute Meige's it is used to prevent recurrence of disease pterygium following its excision compression of the facial nerve by it has an antifibrotic effect which is the inferior cerebellar arteryis a proportional to the concentration recognised cause used stripping of the orbicularis oculi is its use in trabeculectomy is curative. associated with a higher incidence of hypotonyas compared with the use of 5-FU.

In stem cell deficiency: corneal cells at the limbus are absent The following are true about adenovirus keratoconjunctivitis: the cornea is covered bycells membranous conjunctivitis causing resembling those of conjunctiva puntal occlusion is a complication impression cytologyreveals goblet the keratitis is caused byviral cells replication within the epithelial cells corneal oedema is a common subepithelial infiltratestypically feature develop about 2 weeks after the onset of conjunctivitis amniotic membrane graft is used to treat this condition. corneal neovascularisation is a late complication

systemic acyclovir is useful in the acute stage. 90

Essential blepharospasm is Mitomycin C is isolated from characterised bybilateral orbicularis Streptomycescaespitosus and acts as oculi and facial spasm. It usually an alkylatingagent which inhibits begins as excessive blinking and may DNA synthesis.In ophthalmology, it lead to functional blindness.The is used mainly to prevent pterygium cause is unknown. Symptoms usually recurrence and in high-risk improve during sleep and are trabeculectomies. Mitomycin C is a worsened under stressful more potent anti-metabolitethan circumstances. Meige's disease is 5-FU and its use in glaucoma is essential blepharospasm with facial isolated,with an increased risk of grimace, mouth retraction and wound leak and hypotony. retrocollic spasm. It suggests a dysfunction of the basal ganglia. Compression of the facial nerve by the inferior cerebellar arteryis a recognised cause in hemifacial 91 spasm. Treatment is usually with botulinum injection into the Adenoviral keratoconjunctivitis can orbicularis oculi. In resistant cases, occasionally give rise to severe stripping of the orbicularis oculi may membranous conjunctivitis with be effective. punctal occlusion and symblepharon. The keratitis seen is caused byviral replication within the epithelium. With time, the keratitis progresses to subepithelial infiltration which is likelyto be an immune response.Corneal neovascularisation In stem cell deficiency,the cells at is not a feature of this condition. the limbus are absent or defective. Systemic acyclovir is of no use. The result is conjunctivalisation of In patients with severe the corneal epithelium. Goblet cells keratoconjunctivitis, or those with may be present.Corneal oedema is reduced vision from the keratitis, not a feature because the steroids may be used with caution. endothelial function remains normal. Autograft transplant of limbal cells from the fellow eye,allograft transplant from a cadavertic eyeor a living related donor, or amniotic membrane transplant has been used to treat stem cell deficiency. 92 In vitamin A deficiency: 94 The following mucopolysaccharidoses often the blood film often shows give rise to corneal clouding: characteristic features Hunter's syndrome visual loss is usually due to retinal degeneration Hurler's syndrome lipid components of tear are usually affected early Morquio's syndrome Bitot's spots are found onlyin the temporal bulbar conjunctiva Sanfilippo's syndrome nyctalopiausually precedes xerophthalmia. Scheie's syndrome.

93 Steven-johnson's syndrome:

is most often caused bydrugs 95 In keratoconus:

the onset is usually in childhood is characterised byformation of blisters on the skin progressive astigmatic myopia is the may be confined to the skin most common refractive change

patients should avoid contact sport is commonly associated with due to the risk of corneal perforation mucopurulent discharge from the eyesin the acute stage hydrops is an indication for early corneal graft rarelycauses long-term ocular sequelae. corneal graft has a good success rate. 92 94

The blood film is normal in vitamin A Mucopolysaccharidoses are a group deficiency.Abetalipoproteinaemia, a of lysosomalstorage diseases which rare recessive disorder, is associated are inherited in an autosomal with vitamin A deficiency and the recessive fashion with the exception presence ofastrocytic red blood cells of Hunter's syndrome which is in the blood film.Visual loss is X-linked.Although all of them can usually caused bykeratomalacia give rise to corneal clouding through leading to perforation.Dry eyeis deposition of accumulated often due to abnormal mucous metabolites, corneal clouding is only membrane with goblet cell occasionally seen in Sanfilippo's dysfunction.Night blindness syndrome and Hurler's syndrome. (nyctalopia)occurs before xerophthalmia.

95

93 Keratoconus is an ectatic condition characterised byprogressive thinning Steven-johnson's syndrome refers to of the cornea. The condition usually acute inflammation of the skin and begins at adolescence. Progressive mucous membrane. Lesion confined astigmatic myopia is the most to the skin is called erythema common finding.Despite the multiforme. The skin lesion typically thinning, perforation is rare. Hydrop appears as target lesions. results from a break in Descemet's Mucopurulent ocular discharge is membrane leading to cornea common at the acute stage.Ocular swelling.With time, hydrops usually complications are common and resolves, leaving behind a small scar. consist of dryeyes, symblepharon Compared with other inflammatory and eyelidmalposition. conditions, corneal graft in keratoconus has a higher success rate. 96 A man develops lefttrochlear nerve 98 The following are true about palsyfollowing head injury.He is divisional oculomotor nerve palsy: likelyto experience increased verticaldiplopia in the following it is more common than infranuclear situations: third nerve palsy

right gaze it suggests a lesion in the cavernous sinus

head tiltto the right superior division involvement is associated with hypotropia

down-gaze superior division involvement is associated with poor accommodation near reading inferior division involvement causes mydriasis. up-gaze.

99 The following association with the respective eyemovements are 97 In a patient with Horner's syndrome, correct: the following signs are seen: see-saw nystagmus - lesion in the decreased intraocular pressure chiasm

down beat nystagmus - ptosis which improves with Arnold-Chiari malformation phenylephrine ocular flutter - neuroblastoma enophthalmos measurable with an exophthal mometer convergence retraction anisocoria which is most noticeable nystagmus - mid-brain lesion in bright light pendular nystagmus - unilateral problem with accommodation. congenital cataract. 96 98

Following fourth cranial nerve palsy, Divisional oculomotor nerve palsyis vertical diplopia is usually worse on less common than complete third contralateral gaze(in this case right nerve palsy.It suggests a lesion in the gaze),head tiltto the same (left)side, anterior cavernous sinus. Superior down-gaze (such as walking division involvement causes ptosis downstairs) and near reading. and hypotropia,whereas inferior division involvement causes exotropia,hypertropia, intorsion and mydriasis. 97

In Horner's syndrome, the loss of sympathetic tone will decrease 99 intraocular pressure. Ptosis improves with the sympathomimetic effect of See-saw nystagmus is indicative phenylephrine. Enophthalmos is an of a lesion in the chiasmal region. apparent sign rather than a true one, Arnold-Chiari malformation can due to updrawing of the lower lids. present with down beat nystagmus. Anisocoria is typicallymore Opsodonus but not ocular flutter noticeable in dim light. may be a sign of neuroblastoma. Accommodation is normal. Convergence retraction nystagmus is a feature of mid-brain lesion such as pinealoma. Pendular nystagmus is a common feature in patients with poor vision due to congenital cataract, and this can be unilateral. 1 In a patient with proliferative 1 The following are true about ciliary diabetic retinopathy,vitrectomy is bodyablation: recommended in: the procedure is painless an onlyeye with tractional retinal detachment nasal to the optic disc it increases uveoscleral outflow of the presence of bilateral pre-retinal aqueous haemorrhages it decreases aqueous production by the presence of neovascularisation in the ciliary body the inferior arcade with pre-retinal haemorrhage, despite pan-retinal it is not effective in neovascular photocoagulation glaucoma

vitreous haemorrhage of more than sympathetic ophthalmia has been 4 months duration in typeI diabetes reported as a complication.

simultaneous presence of optic disc and peripheral neovascularisation with pre-retinal haemorrhages. 1 In a newborn with a cloudy cornea, the differentialdiagnoses should include:

1 The following contraindications are sclerocornea correct:

systemic acetazolamide in patients congenital hereditary endothelial who develop Steven-Johnson' s dystrophy syndrome to sulphonamide Peter's anomaly topical timolol in patients on peripheral calcium channel blockers granular dystrophy apraclonidine in patients taking monoamine oxidase inhibitors megalocornea. latanoprost in patients with skin melanoma

pilocarpine in patients with raised intraocular pressure secondary to uveitis. 1 1 vqgJ~.•.'."...• '•...•..•.••.• "

The indications for vitrectomy in Ciliarybody ablation involves the use diabetes have been outlined in the of a cryoprobe or a laser to destroy Diabetic Retinopathy Vitrectomy the ciliarybody, thereby reducing Study.These include tractional aqueous production. The procedure retinal detachment involving the can be painful and sympathetic fovea, non-clearing vitreous ophthalmia has been reported as a haemorrhage especiallyin typeI complication. Overtreatment can diabetes,and progressive lead to hypotonyand, rarely,phthisis fibrovascular proliferationdespite may develop.It is the treatment of pan-retinal photocoagulation. choice in patients with poor visual prognosis,with high intraocular pressures (such as neovascular glaucoma), inadequatelycontrolled with medication and/or surgery. 1

Acetazolamide is a sulphonamide derivative and can cause Steven• 1 Johnson's syndrome in patients with ....•... ..•. ~..•..'.''.'....' '.'.' ...•. '...' .. :.'.'.' ,.•...•...... •.... reaction to either drug. Topical ~ timolol is onlycontraindicated in the Granular dystrophydoes not occur presence of central-acting calcium in the newborn but develops later. channel blockers. The concurrent use Megalocornea occurs when the of both can lead to heart block. cornea in the neonate is 12mm or Apraclonidine may cause a more in diameter - it is associated hypertensive crisis in patients taking with corneal cloudiness. A useful monoamine oxidase inhibitors. mnemonic for neonatal corneal Latanoprost can increase melanin opacity is STUMPED: Sclerocornea, production in the iris but has not Tear in Descemet's membrane been shown to affectskin melanoma. (congenital glaucoma or trauma), Pilocarpine can exacerbate the Ulcer (infections),Metabolic breakdown of the blood aqueous (mucopolysaccharidoses), Posterior barrier and is contraindicated in corneal defects (posterior the uveitic patient. keratoconus or Peters' anomaly), Endothelial dystrophies (congenital hereditary stromal, posterior polymorphous dystrophyor congenital hereditary endothelial dystrophy),Dermoid. 104 The following are true about 106 The following are true about normal intraocular pressure measurement tension glaucoma: with the applanation tonometer: brain scan reveals a cause in about following LASIK surgery,the lOP 10% of patients reading may be inaccurately high optic nerve meningioma is a cause of excessive fluorescein may give an normal tension glaucoma inaccurately high reading a dense paracentral scotoma pressure may be falselyelevated in encroaching upon fixationis a the presence of increased corneal common finding thickness the use of betaxolol has been shown myopia may give an inaccurately low to delayprogression pressure measurement due to decreased rigidityof the eyeball a reduction of more than 30% of intraocular pressure has been shown breath holding may give a low to delaythe progression of visual pressure reading. field defect.

105 The following are true about 107 The following are true about Down's perimetry: syndrome: the peripheral visual field is better genetic analysisinvariably shows tested with Goldmann perimetry trisomy 21 rather than Humphrey perimetry discrete lens opacities in the anterior both cataract and miosis can cause and posterior cortex are common decreased sensitivity Brushfield's spots are more short-term fluctuation is used in commonly seen in blue than brown Humphrey perimetry to test the irides reliabilityof the subject hypermetropia is a common finding a contracted visual field is encountered ifthe correcting lens is placed too far from the eye keratoconus with a tendency to developing hydrops is a recognised a normal visual fieldexcludes the association. presence of glaucoma. 1 1

With applanation tonometry, By definition,normal tension increased corneal thickness gives an glaucoma is diagnosed after inaccurately high reading, whereas excluding other causes. Optic nerve decreased thickness gives an meningioma causes compressive inaccurately low reading such as optic neuropathy. Brain scan seldom following LASIK. Myopia does not reveals a cause in most cases of affect the reading with an normal tension glaucoma. A dense applanation tonometer as with an paracentral scotoma encroaching indentation tonometer. Excessive upon fixationis a common finding. fluorescein causes wide mires which Although betaxolol has been shown may give falselyhigh readings.Breath to be neuroprotective in animal holding increases the episcleral models, there has been no venous pressure and may result in a convincing evidence which proves higher pressure measurement. its use in delayingprogression in normal tension glaucoma. The Collaborative Normal Tension Glaucoma Study found that reducing lOP bygreater than 30% delayed 1 the rate of visual field progression.

The Goldmann perimetry covers a wider visual fieldthan Humphrey fieldtests which onlytest the central 30 degrees of visual field.Decreased 1 sensitivityoccurs in cataract, miosis, fatigue and old age. Short-term Down's syndrome is often caused by fluctuation analysesthe response of trisomy 21 but in a small minority it the patient to the same stimulus at can result from chromosomal the same location to assess translocation without an increase in reliability.The visual field is chromosome numbers. Cataract contracted if the correcting lens is with snowflake opacities in the placed too far in front of the cortex, keratoconus with a tendency patient's eye.Glaucoma may still be to developing hydrops,high myopia present despite a normal visual field and esotropia are associated with because visual field defects occur Down's syndrome.Brushfield's spots onlyin the presence of significant are made up of aggregates of iris nerve fibre loss. stromal fibres.They are more commonly seen in blue irides than brown irides. 1 In aniridia: 110 The following are true about the findings of the Diabetes Control and there is complete absence of iris Complications Trial (DCCT): structure good diabetic control in typeII poor vision usually results from diabetes reduces the risk of diabetic excessive photophobia retinopathy

limbal stem cell deficiency with hypoglycaemia is common in corneal neovascularisation is a patients on intensive diabetic feature treatment

Wilm's tumour is most commonly good diabetic control reduces seen in sporadic cases diabetic nephropathy by50%

lens subluxation is a feature. there is no significant difference in the incidence of diabetic neuropathy between patients on standard and intensive diabetic treatment

the risk of cardiovascular diseases is 1 In a patient with peripheral bone• reduced in patients on intensive spicule pigmentation, the following diabetic treatment. tests are useful:

serum ACE concentration 111 In an infant with poor vision, initial constriction of pupils to darkness serum phytanic acid occurs in:

Tay-Sachs' disease plasma ornithine level

achromatopsia plasma lipoprotein B

Leber's congenital amaurosis urine glycosaminoglycans.

X-linked congenital stationarynight blindness

optic atrophy. 1 110

In aniridia,there is a remnant of iris The DCCT is a clinical study tissue.The main cause of poor vision conducted from 1983to 1993by the is foveal hypoplasia.Other associated National Institute of Diabetes and features include glaucoma and limbal Digestive and Kidney Diseases.The stem cell failure with an increased risk study showed that keeping blood of corneal neovascularisation. Wilm's glucose levels as close to normal as tumour is most often seen in familial possible delaysthe onset and cases, especiallyin those with progression of eye,kidney and nerve abnormalities of chromosome 11. disease caused bydiabetes. In fact, The zonules are weak and lens it demonstrated that any sustained subluxation is common. lowering of blood glucose helps, even if the person has a history of poor control. The study involved onlytype I diabetics and did not look at cardiovascular risks. 1

Serum ACE is useful for sarcoidosis but bone-spicule pigmentation is not seen in sarcoidosis.Serum phytanic 111 acid is raised in Refsum's disease. Elevated ornithine level occurs in In Tay-Sachs' disease and optic gyrateatrophy which is atrophy,the pupils dilate in response choroidoretinal atrophy.Plasma to darkness. In Leber's congenital lipoprotein B is absent in amaurosis, pupillary response is abetalipoproteinaemia,a cause of sluggish but the pupil still dilates bone-spicule pigmentation. Urine in response to darkness. glycosaminoglycans are elevated in In achromatopsia and X-linked mucopolysaccharidoses such as congenital stationarynight blindness, Hunter's or Hurler's syndrome, in there is an initial constriction of which bone-spicule pigmentation pupils to darkness. is a feature. 112 In juvenile chronic arthritis, the risk 114 The following are true about of developing uveitis is highest if: neurofibromatosis:

the age of onset is less than 6 years Lisch's nodules are composed of of age melanocytic hamartoma

the gender is female Lisch's nodules are diagnostic of type I neurofibromatosis

serum anti-nuclear antibodies are bilateral acoustic neurofibromas are positive more common in typeI than typeII neurofibromatosis more than four jointsare affected pulsatile exophthalmos is a feature of typeI neurofibromatosis HLA-B27 is positive. posterior subcapsular cataract is a feature of typeII neurofibromatosis.

113 In congenital nystagmus: 115 In sarcoidosis: vision decreases on convergence serum ACE concentration is a good reflection of the activity of the the intensityof nystagmus increases disease with fixation pulmonary function test shows an nystagmus is enhanced when one obstructive pattern eyeis covered Mantoux test is typicallypositive in absence of nystagmus in the dark is those with previous BCG vaccination a feature band keratopathy usually results the vertical optokinetic response is from recurrent anterior uveitis normal. neurosarcoidosis is more common in the presence of retinal periphlebitis. 112 114

Uveitis develops in about 20% of Both typeI and type\I patients with juvenile chronic neurofibromatosis are autosomal arthritis.The risk is highest in the dominant in inheritance. Lisch's female gender, patients less than nodules occur in typeI 6 years of age, pauci-ariticular joint neurofibromatosis and are composed involvement (fewer than four joints of melanocytic hamartoma. Pulsatile affected)and the presence of exophthalmos is seen in typeI positive anti-nuclear antibodies. neurofibromatosis caused bythe absence of the lesser wing of the sphenoid bone.Type \I neurofibromatosis is characterised by acoustic neuroma which may be 113 bilateral.Posterior subcapsular cataract is also a feature. In congenital nystagmus, vision improves on convergence and therefore vision is always better for near than distance. The intensityof the nystagmus increases with 115 fixation.Latent nystagmus is common and closing one eyemay Serum ACE concentration is often enhance the nystagmus. Congenital elevated in active sarcoidosis and its nystagmus is absent in sleep but concentration reflects the activity of present in the dark when both the disease.Pulmonary fibrosisis a eyesare open.There is an inverse complication giving a restrictive horizontal optokinetic response but pattern on pulmonary function test. the vertical optokinetic response Skin anergy is common and the is normal. Mantoux test may be negative in those with previous BCG vaccination. Band keratopathy usually results from hypercalcaemia. The presence of retinal periphlebitis increases the risk of neurosarcoidosis. 116 In myasthenia gravis: 118 Intraocular foreign bodies made up of the following are relativelyinert: the Tensilon test causes ventricular tachycardia aluminium

tendon reflexes are reduced gold

anti-acetylcholine receptor antibodies are more commonly copper detected in generalised disease than ocular myasthenia silver application of ice to a ptotic lid will reduce the ptosis platinum. excision of the thymus should be carried out in patients with ptosis not responding to anticholinesterase or steroid.

119 The typesof giant cells seen in the following conditions are correct:

117 In unilateral facial nerve palsy: chalazion - foreign bodygiant cells

corneal sensation is reduced sarcoidosis - Langhans' giant cells

ipsilateral absence of taste in the anterior two-thirds of the tongue juvenile xanthogranuloma - Touton suggests a lesion in the facial canal giant cells

the eyeis not involved if the lesion tuberculosis - foreign bodygiant involves the upper motor neurone cells

there is retraction of the ipsilateral xanthoma disseminatum - Touton upper eyelid giant cells.

there is a higher chance of recovery for Ramsay Hunt's syndrome compared with Bell's palsy. 116 118

The T ensilon test can give rise to Gold, silver and platinum are bradycardia,thus intravenous relativelyinert and cause little atropine should be readilyavailable reaction. Aluminium causes a during the test.Tendon reflexes are variable amount of reaction. Copper normal in myasthenia gravis. is toxic and in high concentration Although antibodyto acetylcholine (85%or above) may cause a sterile receptors may be detected in about suppurative endophthalmitis. 80to 90% ofgeneralised myasthenia gravis, only50% of pure ocular myasthenia is positive for this antibody.Ice packs can transiently improve muscle activity and 119 therefore alleviate ptosis.Excision of the thymus is not recommended in There are three typesof giant cells: pure ocular myasthenia gravis unless • Langhans' giant cells are seen in a thymoma is present. tuberculosis and sarcoidosis • foreign bodygiant cells are seen in chalazion • Touton giant cells are seen in 111 juvenile xanthogranuloma, xanthoma disseminatum and Erdheim-Chester's disease. The trigeminal nerve is responsible for corneal sensation.Absence of taste suggests that the lesion is in the geniculate bodywhich is located in the middle ear.In upper motor neurone lesion of the facial nerve, the upper face including the eyeis not affected,due to bilateral innervation of the upper face. Retraction of the levator results from the unopposed action of the paralysedorbicularis oculi. Facial nerve recovery is poor for Ramsay Hunt's syndrome but excellent in Bell's palsy. 1 Orbital rhabdomyosarcoma: 1 In internuclear ophthalmoplegia:

has a peak incidence at 10 years of multiple sclerosis is the most age common cause in young people

commonly arises from the medial diplopia is not a problem rectus

is most commonly of the alveolar patients usually complain of type problems with close work

contains cells with a high the lesion is ipsilateralto the eye nuclear-cytoplasmic ratio with abducting nystagmus

usually presents with fever, making it involvement of the sixth nerve indistinguishable from orbital nucleus is common. cellulitis.

1 In peribulbar anaesthesia: 1 Cyclosporin: the patient should be instructed to is insoluble in water look up to avoid perforation of the globe

is excreted unchanged bythe the superonasal quadrant should be kidneys avoided because of the risk of retrobulbar haemorrhage depresses CD4+ but not CD8+ lymphocyte function the risk of complications is lower with a longer needle should be avoided in patients with anaemia as it can affect the risk of complications is lower haematopoiesis with a fine needle

effect can be increased with the globe perforation causes increased concurrent use of ketoconazole. vitreous pressure. 1 122

The most common age of Internuclear ophthalmoplegia may presentation for orbital be unilateral or bilateral.In young rhabdomyosarcoma is 7-8years of patients,the most common cause is age.It commonly arises from multiple sclerosis, whereas in the mesenchymal tissue external to the elderly,cerebrovascular accidents . Occasionally are more common. Rarely,tumour the tumour may arise from the may be the cause. There is failure of conjunctiva. There are three typesof adduction in one eyewith abducting orbital rhabdomyosarcoma: nystagmus in the fellow eyebut embryonal,alveolar and convergence is normal. Diplopia differentiated.Histologically, occurs when the patient looks the tumour cells have a high sideways due to the adduction nuclear-cytoplasmic ratio with little deficit.Close work does not usually cytoplasm.As a result, special tests pose a problem because such as immunostaining or electron• convergence is normal. The location microscopy are needed for diagnosis. of the lesion is in the medial The tumour may be misdiagnosed as longitudinal fasciculus ipSilateral to orbital cellulitis, but unlike cellulitis, the eyewith the adduction deficit. fever is absent. Involvement of the sixth nerve nucleus constitutes the one-and-a-halfsyndrome.

1

Cyclosporin is now commonly used 123 as second-line treatment in patients with severe uveitis. It is also used in For safe peri bulbar anaesthesia, the patients with high-risk corneal graft. patient should keep the eyein the It is lipophilic and insoluble in water. primary position.The superonasal It depresses the function of CD4+ and inferonasal quadrants contain a lymphocytes but has no effect on rich vascular supply and injection CD8+ lymphocytes. Compared with into these sites may be associated other immunosuppressants, it does with an increased risk of retrobulbar not affect bone marrow function. Itis haemorrhage. The use of shorter and metabolised bythe liver and its finer needles helps to minimise effect is enhanced in the presence of complications through injuries of the P450 inhibitors such as ketoconazole. nerves and vessels.Globe perforation results in hypotony. 1 The following are true about 1 In a patient presenting with dendritic posterior capsular thickening: ulcers, the following suggest that the lesion may not be caused byherpes it occurs in 50% of post-cataract simplex: patients over a 5-yearperiod presence of uveitis it is more common in younger patients absence of sensation it is more common in patients with diabetic retinopathy absence of terminal bulbs it is caused bymultiplication of the epithelial cells presence of ring infiltrates it can be prevented byvigorous polishing of the posterior capsule. staining with rose bengal.

1 The following are recognised features of ophthalmic herpes zoster: 1 The pairings of the following head haemorrhagic glaucoma posture and the respective ocular motilitydisorders are correct:

myositis right sixth nerve palsy- face turn to the right

par planitis right fourth nerve palsy- head tiltto the right

ocular hypertension in the presence bilateral fourth nerve palsy- chin of anterior uveitis depression

diffuse iris atrophy. bilateral superior rectus palsy- chin elevation

chronic progressive external ophthalmoplegia - chin elevation. 1 1

Posterior capsule thickening can Most dendritic ulcers are caused by significantlyreduce visual acuity.It is herpes simplex.Two conditions that usually caused bymigration of the can mimic herpetic dendrites are epithelial cells to the posterior pole herpes zosterkeratitis and early and multiplication of the epithelial stages of acanthoamoeba. In both cells. It is more common in younger cases the characteristic terminal patients,in diabetic retinopathy and bulbs of herpetic dendrites are uveitis. Polishing of the posterior absent.In acanthoamoeba, the pain capsule does not prevent opacity is usually disproportionate to the from developing, as epithelial cells clinical appearance of the lesion. can still migrate from the anterior Other features of acanthoamoeba capsule. keratitis include ring infiltratesand perineural infiltrates.Decreased corneal sensation,variable amount of uveitis and staining of the lesion with rose bengal are common

1 ..•.....•.•..••...•..•...... •...~. features of herpetic dendritis. •....',«:_-s

Follicular conjunctivitis and episcleritis are common in patients with ophthalmic herpes zoster. Intraocular complications are most 1 common in those with nasociliary involvement. These can include Abnormal head posture is adopted keratitis,uveitis, vasculitis and bypatients to maintain binocular myositis.Uveitis may be associated single vision.In a sixth nerve palsy, with hypopyonand haemorrhage; the face is turned in the direction of trabeculitis is common and may give the affected eye.In a fourth nerve rise to troublesome glaucoma. Iris palsy,the head tiltis opposite to the atrophy can occur as a result of side of the lesion.Chin depression is vasculitis and this is typically seen in bilateral fourth nerve palsy sectorial rather than diffuse. and less commonly in bilateral inferior rectus palsy.Chin elevation occurs with ptosis as in chronic progressive external ophthalmoplegia and superior rectus palsy. 128 The following are true about ptosis 130 Benign intracranial hypertension: surgery: is a complication of treatment for in involutional ptosis,levator acne resection is seldom required does not give rise to focal in a patient with right amblyopiaand neurological deficits an ipsilateralcongenital ptosis covering the pupil, unilateral brow typicallyshows dilated ventricles on suspension is the treatment ofchoice the CT scan

in ptosis due to chronic progressive requires optic nerve fenestration to external ophthalmoplegia, levator be performed prophylacticallyto resection is the treatment of choice prevent optic nerve damage

ptosis surgery should never be is unlikely to be the diagnosis ifthere performed on patients with are hard exudates at the macula. myasthenia gravis

in Marcus-Gunn jaw-winking ptosis, unilateral lid surgery is usually unsatisfactory.

129 In sickle cell disease:

HbSC and HbSThal are more likelyto cause proliferative retinopathy than HbSS

degenerative retinoschisis is commonly seen in the superotemporal quadrant

an increased foveal avascular zone with microaneurysms is a cause of poor vision

proliferative vitreoretinopathy most commonly affects the peripheral retina

blood transfusion reduces the severityof the retinopathy. associated with severe proliferative 1 .....••...•••••...;.,.•..••.•••••.•••.•. i.•.•.• :. •.;~;:;~;p;: retinopathydespite a benign systemic course. Retinoschisis occurs as a The choice of ptosis surgery depends on the function of the levator. result of resorption of intraretinal In involutional ptosis,the levator haemorrhage. Occlusion ofcapillaries function is good and therefore only causes an increased foveal avascular advancement is required. In the zoneand microaneurysms may be presence of amblyopia and severe seen, especiallywith fluorescein ptosis,unilateral brow suspension is angiography.Proliferative retinopathy is an important cause of visual loss usually not satisfactorybecause the patient may not use the frontalis through vitreous haemorrhage and retinal detachment. The new vessels muscle to liftthe eyedue to usually occur in the peripheral retina. amblyopia.The choice of surgery is Blood transfusion does not affect the disinsertion of the contralateral eye retinopathy. followed bybilateral brow suspension. In chronic progressive external ophthalmoplegia, levator resection may lead to corneal exposure when the function of the 130 orbicularis oculi weakens with time. Brow suspension is the Benign intracranial hypertension recommended technique. Although typicallyaffects females who are myasthenia gravis can cause ptosis, obese.Patients on tetracycline or persistent ptosis may develop which steroid may also develop this is unresponsive to systemic condition. The diagnosis is made cholinesterase treatment. Surgery after excluding other causes of raised may be performed provided the intracranial hypertension.There ptosis has been recorded as should be no focal neurological being stable for about 1 year. deficit,although false localising signs In Marcus-Gunn jaw-winking such as sixth nerve palsymay be syndrome, unilateral ptosis surgery present.Optic nerve fenestration is does not abolish the winking. indicated in progressive visual field Bilateral levator disinsertions loss despite repeated removal of the followed bybrow suspension is cerebrospinal fluid.Macular exudate recommended. may develop, causing visual disturbance. 1

In sickle cell disease,HbSC and HbSThal are most commonly 131 The following are true about the 1 In Marfan's syndrome: management of cytomegaloviral (CMV) retinitis: blood test is useful for diagnosis

zidovudine (AZT) is not effective against CMV retinitis there is a point mutation in the long arm of chromosome 15 the main complication ofgancyclovir is bone marrow toxicity the majorityof cases are the result of a new mutation the main complication offoscarnet is nephrotoxicity lens subluxation is in the superonasal direction viral resistance to gancyclovir will also prevent the effective use of myopia is common. foscarnet

intravitreal administration of gancyclovir or foscarnet will avoid systemic complications. 1 In a patient with sudden onset of visual loss,the following favour the diagnosis of ophthalmic arteryover central retinal arteryocclusion: 1 Glycosylated haemoglobin assay: absent a- and b-wave on the ERG reflects red blood cell exposure to glucose in the last six weeks absent cherry red spot is used in the diagnosis of diabetes mellitus relative afferentpupillary defect correlates with the risk of development of microvascular disease visual acuity of light perception

is artificiallyhigh in a patient with sickle cell anaemia delayedchoroidal circulation during fluorescein angiography. if low in diabetes mellitus can be associated with hypoglycaemia. 131 1

AZT is not effective against CMV Marfan's syndrome is a clinical retinitis.The three drugs most diagnosis.The point mutation is in commonly used are gancyclovir, the fibrillingene on the long arm of foscarnet and cidofovir.Gancyclovir chromosome 15.The majorityof causes bone marrow toxicity, cases are inherited and only15% are whereas foscarnet and cidofovir caused bynew mutations. Ocular cause nephrotoxicity.Viral resistance manifestations include myopia and to one drug does not prevent the use ectopia lentis with superonasal of another to be effective.In patients subluxation of the lens. with systemic complications, intravitreal drugs can be given.

1

1 In ophthalmic arteryocclusion, the visual loss is usually profound and Glycosylated haemoglobin reflects vision is typicallylight perception or red blood cell exposure to glucose in worse, as opposed to counting finger the last six weeks. It is used to assess vision in central retinal artery diabetic control. It is not used to occlusion. As the choroidal diagnose diabetes mellitus. It circulation is affected,cherry red correlates well with the risk of spot is absent.The ERG in central microvascular disease.In sickle cell retinal arteryocclusion shows a anaemia, the concentration may be decreased b-wave amplitude and a artificiallylow. A low level in a relativelynormal a-wave; in diabetic may indicate that the ophthalmic arteryocclusion, both patient is predisposed to recurrent a- and b-wave are absent.Relative hypoglycaemia. afferentpupillary defect is present in both conditions but is usually more marked in ophthalmic artery occlusion. 1 In a patient with proptosis,the 1 In carotid arterystenosis: following features favour a lesion in the orbital apex rather than the the atheroma is most commonly cavernous sinus: found at the bifurcation of the common carotid artery decreased vision with relative afferentpupillary defect the bruit, if present, is best heard opposite the thyroid gland abduction deficit the risk of stroke is higher with cerebral TIAs (transientischaemic dilated pupil attacks) than with ocular TIAs

endarterectomy is superior to absent forehead sensation medical treatment in the prevention of stroke in a patient with moderate stenosis loss of sensation in the lower eyelid. the addition of dipyridamoleto aspirin has no additive effect in reducing the risk of stroke.

1 The following are true about photodynamic therapy:

it is effective in both classic and occult choroidal neovascularisation

indocyanine green angiography is essential before commencement of treatment

retinal pigment epithelial detachments are not suitable for treatment

therapy works bycausing proliferationof the retinal pigment epithelium

argon laser is used. 1 137

The optic nerve does not pass Carotid artery stenosis can give rise through the cavernous sinus and to recurrent TIAs and stroke.The therefore the presence of a relative stenosis caused byan atheroma is afferentpupillary defect suggests a found usually at the bifurcation of lesion in the orbital apex. the common carotid artery.The bruit An abduction deficit is caused bya is best heard over the angle of the sixth nerve palsy,a dilated pupil by jaw.The annual risk of stroke is 8% a third nerve lesion and absent for cerebral TIAs and 2% for ocular forehead sensation byan ophthalmic TIAs. However, the cause of death is nerve lesion.All the aforementioned usually myocardial infarction rather structures pass through both the than stroke. Endarterectomy is cavernous sinus and the orbital apex. superior to medical treatment in Loss of sensation in the lower eyelid symptomatic cases with significant is caused bymaxillary nerve stenosis of over 70%. involvement and suggests a lesion Endarterectomy has not been shown in the cavernous sinus because to be advantageous over medical this nerve does not enter the treatment in moderate stenosis of orbital apex. between 50 and 60%.Aspirin is the medication of choice. Dipyridamole can be given as an alternative but has no additive effect. 1

In photodynamic therapy,a photosensitive dyesuch as verteporfin is injected which is then taken up bythe endothelium of the choroidal neovascularisation. Infra-redlaser is used to irradiate these vessels causing damage to the endothelium but sparing the retinal pigment epithelium and the photoreceptors. Only classic choroidal neovascularisation is suitable for treatment; patients with haemorrhage and retinal pigment epithelial detachment are not suitable for treatment. 1 With regard to the following visual an active herpetic lesion of the field defects: cornea is a contraindication to surgery a right temporal lesion causes a left superior homonymous a should be quadrantanopia performed in the presence of a nasolacrimal duct obstruction prior a leftsuperior occipital lesion causes to cataract surgery. a right inferior homonymous quadrantanopia

a junctional scotoma comprises a central scotoma in one eyeand 1 The following are true of superotemporal fieldloss in the other endophthalmitis which developed 2 years post-trabeculectomy: a junctional scotoma is a feature of Kennedy-Foster's syndrome it does not occur in the absence of a leaking bleb bitemporal hemianopia from a pituitarylesion usually begins it is associated with the use of superiorly. 5-fluorouracil

it is most commonly caused by Staphylococcus

1 The following are true when it has a benign course compared assessing patients for cataract with other forms of post-operative surgery: endophthalmitis

patients with Fuch's heterochromic it can be effectivelytreated with cyclitis should onlyundergo surgery intensive topical antibiotics. if there is an absence of cells in the anterior chamber for at least one week

patients with significant diabetic macular oedema should undergo laser photocoagulation before cataract surgery

patients with lattice degeneration and atrophic holes should undergo cryotherapy before cataract surgery 1 oedema should undergo laser ,.·..•.' ...•. '.·.a.· ..'.·.·.'.·.·;.·.· .•.;.•. ·'., ~.· ·...•...•..•.""· ,',~,-,---/'.e...... ·..•....•. '.,, ~ e treatment prior to cataract surgery. A temporal lobe lesion causes a In maculopathy, it is recommended that laser is performed 12 weeks superior homonymous quadrantanopia, whereas a parietal before cataract surgery.Retinal lobe lesion causes an inferior detachment following cataract homonymous quadrantanopia. surgery is usually the result of a fresh retinal tear.Prophylactic treatment Lesion in the occipital lobe causes of lattice degeneration and atrophic homonymous hemianopia, and a holes is not necessary.Active ocular lesion in the lower occipital lobe infections,including herpetic causes superior homonymous keratitis,is a contraindication to quadrantanopia. A junctional cataract surgery.Nasolacrimal duct scotoma is caused bya lesion at the junction between the optic nerve obstruction in the absence of a mucocele is not a contraindication and the optic chiasm, resulting in the to cataract surgery. compression of the knee of Wilbrand. The visual field defect is made up of an ipsilateral central scotoma and a contralateral superotemporal field defect. 140 Chiasmal lesions from a pituitary lesion typicallypresent with early Infection can enter the eyethrough a superior field loss. microscopic break in the absence of an obvious leaking bleb.A thin cystic bleb such as those created with the use of mitomycin C or 5-fluorouracil 1 increases the risk of infection.The most commonly isolated pathogens are and In Fuch's heterochromic cyclitis,cells Streptococcus Haemophilus. The endophthalmitis can cause in the anterior chamber do not severe visual loss unless treated respond to steroid and their early.The management should presence should not result in include a vitreous tap for culture and postponement of cataract surgery. sensitivityand intravitreal antibiotics. However, Fuch's heterochromic cyclitis is associated with certain operative complications due to poor pupillarydilatation and a tendency for the iris to bleed (Amsler's sign). Patients with proliferative diabetic retinopathy and significant macular 1 The following are true about limbal 1 Orbital lymphomas: dermoids: most often arise from the lacrimal they are hamartomas fossa

should not be biopsied due to the they are most commonly found in risk of tumour dissemination the inferotemporal region are of high-grade non-Hodgkins' limbal dermoids are more commonly lymphoma in the majorityof cases associated with systemic diseases than orbital dermoids usually have systemic involvement at presentation limbal dermoids give a plus cylinder with an axis along the meridian of are best treated with radiotherapy. the lesion

earlyremoval of limbal dermoids will prevent refractive amblyopia.

1 The following organisms can penetrate an intact cornea:

1 Acoustic neuromas: Staphylococcusaureus

arise from the vestibular nerve ofthe eighth cranial nerve Pseudomonas aeruginosa

are benign tumours of Schwann cell origin Neisseriagonorrhoea

cause a reduced blink reflex Corynebacterium diphtheriae

commonly present with facial nerve palsy Escherichiacoli.

are a feature of neurofibromatosis typeII. 1 143

Limbal dermoids are choristomas, The lacrimal fossa is the most i.e.proliferation of normal tissue in common location (about 50%)for abnormal locations.They are most orbital lymphoma. Biopsyis essential commonly located in the for diagnosis.The majorityof orbital inferotemporal region.Systemic lymphomas are of low grade and do diseases such as Goldenhar's not show systemic dissemination at syndrome are an association.They presentation.However, long-term often induce oblique astigmatism follow-up is essential because of the with a plus cylinder axis along the risk of systemic involvement. meridian of the lesion.Despite early Radiotherapy is the treatment of removal, the astigmatism usually choice. remains. Some refractive amblyopia is present even with spectacle correction. 1

1 Bacteria that can penetrate an intact .....'.. '.'... '."".' ... '" ~"' ~•.·".·',e.··,,·· ..·.•..•· •••.••.•... "·" cornea include Neisseriagonorrhoea, ~ ® Nesseriameningitidis and Acoustic neuromas, despite the Corynebacterium diphtheriae. name, arise from the vestibular nerve of the eighth cranial nerve. The most common location is the internal auditory canal. The tumour is made up of benign Schwann cells.Acoustic neuromas can cause compression of the fifth or seventh cranial nerve, resulting in a reduced or abnormal blink reflex.Facial nerve palsyis a rare presentation.Most patients present with neurosensory deafness, disequilibrium or tinnitus. 1 The following are true about cataract 1 The following are indications for seen in congenital rubella: surgery in a blowout fracture:

nuclear sclerosis is a feature cosmetically unacceptable enophthalmos

microspherophakia is a feature of fracture involving more than 50% rubella cataract of the floor

the rubella virus can be isolated from diplopia on up-gaze the lens content

retention of lens fibre nuclei is a anaesthesia of the lower lid feature

it is a common cause of glaucoma in hyphaema. congenital rubella.

1 Endothelial proliferationis a feature 1 In a patient with an alkaline burn: of: the eyeshould be irrigated congenital hereditary endothelial immediately until the pH is about 7.5 dystrophy calcium hydroxide (lime)causes iridocorneal endothelial syndrome more damage than sodium hydroxide

chronic uveitis limbal ischaemia is used to decide on the severityof the damage

traumatic angle recession conjunctival erythema is associated with poor visual prognosis

posterior polymorphous dystrophy. topical steroids are contraindicated due to the risk of perforation. 1 141

Bilateral or unilateral cataracts are Repair of the blowout fracture is common in congenital glaucoma. indicated in: The cataract typicallyinvolves the • cosmetically unacceptable nucleus but white cataracts are also enophthalmos commonly seen.The lens tends to be small and spherical. The rubella virus • a large fracture • entrapment of the inferior rectus has been isolated from lens material • presence of diplopia in the years after birth. Cataract extraction primary position. may release the virus, resulting in endophthalmitis. Retention of lens fibre nuclei is common. Glaucoma often co-exists with cataract but is not caused bythe cataract. 148

Immediate irrigation is essential until the pH is about 7.5.Lime causes less 1 damage to the eyethan sodium hydroxide or ammonium hydroxide. Congenital hereditary endothelial The presence of limbal ischaemia is dystrophyis characterised byabsent an indication of the severityof the or atrophic endothelium, resulting in injuryand can predict the extent of corneal oedema. Chronic uveitis stem cell loss.Conjunctival erythema does not cause endothelial is common and does not affect the proliferation.Endothelial proliferation visual prognosis.Steroids should be occurs in iridocorneal endothelial used cautiously due to the risk of syndrome,traumatic angle recession sterile ulceration and, occasionally, and posterior polymorphous perforation. dystrophy.This proliferationcan involve the trabecular meshwork, giving rise to glaucoma. 1 In a child with accommodative 1 The following are true about Nd:YAC esotropia: capsulotomy:

the condition can usually be the laser is absorbed bythe pigment detected at birth in the eye

cycloplegic refraction is required to it can elicit propionibacterium acne avoid under-estimation of the endophthalmitis hypermetropia raised intraocular pressure is higher the amount of hypermetropia is in sulcus fixated lenses than 'in the usually greater than 1.5D bag' lenses

the presence of inferior oblique complications of lens subluxation are overaction does not support the most commonly seen with plate diagnosis haptic lenses

strabismus surgery may be an option pupil block glaucoma is a instead of glasses to correct the complication. esotropia.

1 In a patient with lattice dystrophy: 1 The following are true statements about ocular complications of the lesion usually involves the ionising radiation: central cornea and spares the periphery a radiation has a deeper tissue penetration than f3 radiation the condition usually presents with recurrent erosion corneal syndrome true exfoliationof the lens is a feature the main typeof amyloid is amyloid P corneal perforation can result from dryeye and limbal stem cell failure presence of amyloid within the vitreous is common the retina is more sensitive to radiation than the lens compared with macular or granular dystrophy,recurrence in the corneal radiation retinopathy is caused graft is higher. chiefly bydamage to photo receptors. 1 151

In accommodative esotropia, Nd:YAG laser works bybreaking esotropia results from excessive molecular bonds in tissue. Unlike accommodation in response to high argon laser, Y AG laser is not hypermetropia.The age of onset is absorbed bypigment. In a patient usually between 2 and 3 yearsof age. with sequestered focus of It is rarelydetected before the age of propriobacterium acne in the 6 months. Cycloplegic refraction is capsule, disruption of the capsule essential as accommodation can can give rise to endophthalmitis. underestimate the hypermetropia. Post-VAG capsulotomy-induced Inferioroblique overaction and ocular hypertension is more dissociated vertical deviation can common and is higher in patients occur in accommodative esotropia. with sulcus fixated lenses than those Operating on the muscle does not with 'in the bag' lenses.Lens resolve the esotropia as the eyewill subluxation is uncommon but can still converge when the eyeattempts occur with plate haptic lens.Pupil to accommodate. block glaucoma can occur ifthere is prolapse of vitreous into the anterior chamber.

1

f3 radiation has a deeper tissue 1 penetration than a radiation.True exfoliationis a feature of infra-red Lattice dystrophyis an autosomal radiation which is a form of dominant condition. The lesions non-ionising radiation.Dry eyeand typicallyinvolve the central cornea limbal stem cell failure occur with with clear intervening spaces. The ionising radiation which can lead to periphery is spared.The condition corneal perforation.The lens is more usually presents as recurrent corneal vulnerable than the retina to erosion syndrome.Vision is usually radiation damage. Radiation normal until the 40s.The most retinopathy is the result of common typeof amyloid seen is microangiopathy. amyloid P rather than amyloid A. Most cases of lattice dystrophyhave amyloidosisconfined to the cornea. It has a higher recurrent rate within the graft compared with other forms of stromal dystrophy. 1 True statements about latent 1 The following are true about chronic nystagmus include: primary angle-closure glaucoma:

it is more common than manifest occurs most commonly in a latent nystagmus hypermetropic patient

it is absent when both eyesare open chronic retrobulbar pain is a feature

the fast phase of the nystagmus is cataract extraction often decreases towards the occluded eye the pressure

in the presence of congenital laser iridotomy is not useful esotropia,squint operation usually abolishes latent nystagmus progressive angle closure may occur the amplitude decreases with age. in the presence of a patent peripheral .

1 24 hours post-trabeculectomy: 1 The following are true about gases used in retina surgery: a minus lens can usually improve the vision in patients with shallow an ideal gas is one with low surface anterior chamber tension

an increase in the intraocular expansion of gas is due to diffusion pressure is usually caused by of oxygenand carbon dioxide malignant glaucoma sulphur hexafluoride usually dissolves the presence of choroidal effusion is completely one week after injection associated with increased risk of trabeculectomy failure sulphur hexafluoride is more expansile than perfluoropropane shallow anterior chamber with iris-to-cornea touch does not require nuclear sclerosis is a complication of intervention intravitreal gas usage.

shallow anterior chamber with lens• to-cornea touch requires immediate reformation of the anterior chamber. 1 155

Latent nystagmus is more often seen Chronic angle-closure glaucoma can than manifest latent nystagmus. It is occur in patients with chronic common in congenital esotropia.The open-angle glaucoma. It is more nystagmus is absent with both eyes common in females and open and the fast phase of the hypermetropic patients.Pain is not a nystagmus is towards the occluded feature.The angle closure can be eye.Surgery on the muscle does not abolished with cataract extraction abolish it.With time, the amplitude provided there is no anterior of the nystagmus decreases. synechiae.Laser iridotomyshould be tried in these patients to break the pupillary block. Progressive angle closure may occur in the presence of a patent peripheral iridectomy and 1 repeated periodic gonioscopy is imperative. A shallow anterior chamber causes anterior movement of the lens, resulting in myopia.Malignant glaucoma is an uncommon cause of raised intraocular pressure even after 156 trabeculectomy. One must exclude blockage of the sclerotomy byiris Intravitreal gases are used as volume tissue, failure of filtrationdue to tight replacement after vitrectomy and to sutures or retained viscoelastic. retinal tamponade. The most Choroidal effusion is a common early commonly used gases are air, finding following trabeculectomy sulphur hexafluoride (SF6) and and usually has no long-term perfluoropropane (C3F8).Air implications. Shallow anterior dissolves most rapidly(5 days) chamber with iris-to-cornea touch followed bySF6 (2weeks) and C3F8 is usually a transient phenomenon (2months). Expansion of gas is and needs onlyobservation. caused bydiffusion of oxygen and Lens-to-cornea touch requires early carbon dioxide.Air does not expand, reformation due to the risk of SF6 expands to twice its volume and corneal.decompensation and C3F8 expands byabout 4 times. cataract. Posterior subcapsular cataract is a complication of using intravitreal gas. 1 With regard to viscoelastic 1 Vitritis is a prominent feature in: substances: multiple choroiditis they have the abilityto transform from gel into liquid under pressure acute multifocal placoid punctate healon is a cohesive viscoelastic epitheliopathy (AMPPE)

punctate inner choroidopathy a cohesive viscoelastic has better endothelial protective function progressive outer retinal necrosis capsulorrhexis is easier to perform (PORN) with a cohesive viscoelastic birdshot choroidopathy. a dispersive viscoelastic is easier to insert and remove.

1 The advantages of indocyanine 1 Silicone oil: green (lCG) over fluorescein angiography (FFA) include: is more effective than SF6 in the management of proliferative better visualisation of retinal vitreoretinopathy vasculature

is lighter than water better visualisation of the choroidal circulation

causes abnormal ERG and EOG does not result in skin and urine discolouration

acts as a plus lens in aphakic eyes less nausea after injection

can be used instead of gas to avoid an exciting light source is not posturing. required. 1 159

Viscoelastics have pseudoplasticity Vitritis is often minimal or absent in properties,i.e. the abilityto punctate inner choroidopathy, transform under pressure from gel to progressive outer retinal necrosis and liquid.Viscoelastics can be divided birdshot choroidopathy. into cohesive and dispersive types. Healon is a cohesive viscoelastic and viscoat is dispersive.A cohesive viscoelastic has better shock absorption,provides a better view 160 for capsulorrhexis and is easier to insert and to remove. ICG is highly protein bound and A dispersive viscoelastic gives better therefore does not leak out from the coating and endothelial protection choroidal vasculature. This makes it but gives a poorer view and is more ideal for the study of choroidal difficult to insert and remove. circulation and pathology.It is excreted unchanged bythe liver and does not discolour skin or urine. Nausea is less common compared with FFA. ICG requires an exciting 1 light source like FFA.

Silicone oil is used as a long-lasting volume replacement following vitrectomy. It is more effective than SF6 in the management of proliferative vitreoretinopathy.It is lighter than water and very viscid. The use of silicone oil has been associated with abnormal ERG and EOG which may indicate a certain degree of retinal toxicity.In the aphakic eye,it acts as a plus lens and in the phakic eyeas a minus lens. In those who cannot posture, such as the elderly,silicone oil can be used instead of gas. 1 True statements about par planitis 1 Characteristics of toxoplasmosis include: include:

it is uncommon before the age of 10 most cases of ocular toxoplasmosis result from ingestion of oocytes

multiple sclerosis is a common siblings of patients with ocular association toxoplasmosis rarelydevelop ocular findings involvement of the anterior segment the organism typicallyinvolves the excludes this diagnosis superficial retinal layer macular oedema is the most serum Ig M anti-toxoplasma antibody common cause of poor vision is useful for the diagnosis of ocular systemic steroids are the treatment toxoplasmosis of choice for reducing inflammation. toxoplasma uveitis involving the posterior pole usually resolves spontaneously.

1 Post-operatively,an eyehas a In biometry for intraocular lens refraction of + 1.50D which was 1 originallyplanned to be -0.50D. The calculation: following may be responsible: intraocular lens calculation using traditional keratometry in patients nuclear sclerosis who have had photo refractive surgery will result in post-operative hypermetropia asteroid hyalosis videokeratography is more accurate than manual keratometry in patients littleapplanation during A-scan who have had photo refractive surgery

posterior vitreous detachment error in keratometry measurement has a greater effect than axial length measurement eccentric staphyloma. the velocity of ultrasound is faster in the silicone oil than in vitreous

axial length in a patient with silicone oil will give a falselyhigh value. 1 163

Par planitis is characterised by The majorityof ocular toxoplasmosis vitreous condensation and cellular is acquired through congenital aggregation in the inferior vitreous transmission. Once immunity is body.There are two peaks for its established the subsequent foetuses incidence: 5-15 and 25-35 years of are rarelyinfected. The organisms age.It has no known cause. have a predilection for superficial In chronic cases, there may be a retina.Serum Ig G anti-toxoplasma posterior subcapsular cataract, antibodyis useful for the diagnosis. posterior synechiae and/or band Reactivation of a toxoplasma lesion keratopathy.The main cause of can give rise to self-limitingposterior visual loss is macular oedema. uveitis; unless important structures The treatment of choice for poor or vision are threatened, treatment is vision secondary to macular oedema not necessary. is subtenon corticosteroid.

1 164 ~ (]D • ~..0:';,;:_<:;::' ;.· •...t ••·•· .• ·..•• · .•.·.• ... • ..• •.•.•._:',:_,_,./••.•••.. e..·• •.••••...•... • ..•.•••.....• ...· .... Photo refractive surgery flattensthe A hypermetropic calculation will cornea and can result in result ifthe axial length is falselylong miscalculation of intraocular lens and vice versa. Asteroid hyalosisand power, giving a more hypermetropic posterior vitreous detachment can calculation than expected. give a shorter axial length than Videokeratography assesses more expected byreflecting the ultrasound points on the cornea and provides before it reaches the retina. more accurate keratometric results. Insufficient applanation can create a In biometry,error in axial length fluid space between the scan and the measurement has a greater effect on cornea, giving a falselyhigh axial the result than error in keratometric length. Eccentric staphyloma also measurement. Ultrasonic waves gives falselylong axial length ifthe travel more slowly in silicone oil and ultrasound is not aimed at the therefore give a falselyhigh axial macula. Nuclear sclerosis does not length measurement. affect the biometry. 165 The following are true about A 167 The following changes are pattern strabismus: responsible for involutional entropion: the eyeis less exotropic or more esotropic on up-gaze lateral canthal laxity

it is associated with overaction ofthe superior oblique muscle orbital fat atrophy

it is seen in Crouzon's disease lower lid retractor laxity

it can be treated byanterio-posterior displacement of the inferior oblique shallowing of the inferior fornix muscle

it can be treated byupward spasm of the orbicularis oculi. transposition of the medial rectus muscle.

168 In orbital myositis: 166 With regard to medication used in cataract surgery: an underlying cause can be found in the majorityof patients flurbiprofen can be used to prevent miosis but has no mydriatic effect the most commonly involved muscles are the superior and medial intracameral lignocaine must be recti preservative-free CT scan shows enlargement of the the use of adrenaline in the infusion muscle bellywith sparing of the fluid is contraindicated because of tendon the risk of cystoid macular oedema the pain is worse on looking away acetylcholine is used to constrict the from the action of the affected pupil rapidly muscle

subconjunctival antibiotics have systemic non-steroidal anti• been shown to reduce the rate of inflammatory drugs should be tried post-operative endophthalmitis. before systemic steroid. 1 167

A pattern strabismus occurs when The following changes are believed the eyeis less exotropic or more to•• have contributed to involutional esotropic on up-gaze.It is associated entropion: with overaction of the superior oblique muscles. In craniosynostosis • fat atrophy such as Crouzon's disease or Apert's • laxityof the medial and lateral canthal tendons syndrome,there is often inferior • laxityof the inferior retractor oblique overaction, giving rise to a • over-riding of the pre-tarsal V pattern.Inferior oblique muscle muscle bythe pre-septal muscle. weakening does not improve the A pattern.Transposition of the Shortening of the inferior fornix horizontal recti can be used to causes cicatricial entropion, and reduce the A pattern - this is spasm of the orbicularis oculi causes achieved bymoving the medial spastic entropion. rectus upwards or bymoving the lateral rectus downwards.

1 168 e Orbital myositisis idiopathic in the Flurbiprofen can be used pre• majorityof patients.It is part of operativelyto prevent miosis but it the spectrum of idiopathic orbital has no intrinsic mydriatic effect. inflammatory syndrome.The It can also be used to reduce superior and medial recti are most inflammation if corticosteroids are commonly involved. CT scan shows contraindicated. Any drug used in enlargement ofthe muscle bellywith the anterior chamber should be involvement of the tendon.Pain is preservative-free because worse when the eyelooks away from preservatives can damage the the action of the affected muscle. endothelium. Unpreserved Forced duction test is often positive. adrenaline is added to the infusion Chronic cases can lead to fibrosis solution to prevent miosis during and restriction of ocular movement. phacoemulsification. Acetylcholine is In the acute stage, a non-steroidal used for rapid pupillaryconstriction. anti-inflammatorydrug may Although subconjunctival antibiotics be sufficient to control the are given routinely at the end of inflammation. Ifthis fails,systemic cataract surgery, studies have not steroids are usually required. shown a clear reduction of the endophthalmitis rate. 1 Heterochromic iridis occurs in the 1 In nanophthalmos: following: the palpebral fissure is reduced acquired Horner's syndrome

intraocular structures are Sturge-Weber's syndrome disorganised

microspherophakia is usual Waardenburg's syndrome

the sclera is abnormally thickened Hirschprung's disease

intraocular surgery is associated with naevus of Ota. rhegmatogenous retinal detachment.

1 Regarding Sturge-Weber's syndrome: 1 Latanoprost: the port-wine stain does not cross decreases the formation of aqueous the midline humour glaucoma is more common in a should be avoided in the presence of patient with upper lid rather than anterior uveitis lower lid involvement

causes iris hyperpigmentation by recurrent epilepsyand mental inciting melanocyte hyperplasia retardation are features

should be avoided in pseudophakic tram-line appearance on the plain patients skull X-rayis the result of calcification ofthe abnormal cerebral anti-glaucoma effect is reduced with vessel the concurrent use of a non-steroidal anti-inflammatorydrug. expulsive haemorrhage is a recognised risk during cataract surgery. 1 111

Congenital rather than acquired In nanophthalmos, the axial length of Horner's syndrome is associated with the eyeis reduced. The lens, heterochromia iridis with the however, is of normal sizeand this affected side being lighter. increases the risk of pupillary block. Sympathetic innervation is important The sclera is abnormally thickened for the migration ofmelanocytes into and this is thought to cause the iris.Sturge-Weber's syndrome uveal effusion syndrome and can give rise to ipsilateraliris rhegmatogenous retinal detachment, hyperpigmentation.Waardenburg's especiallyafter intraocular surgery, syndrome causes hypochromia of through vortex vein compression. the iris.Hirschprung's disease results from abnormal sympathetic nervous system development and hence abnormal melanocyte migration. Naevus of Ota causes increased 1 pigmentation of the affected side. Sturge-Weber's syndrome is a type of phakomatoses. It has a sporadic occurrence. The condition is characterised bya port-wine stain 1 involving the trigeminal distribution. However, the lesion often crosses the Latanoprost works byincreasing midline.Glaucoma is a common uveoscleral outflow. It causes blood complication due to increased aqueous barrier instabilityand episcleral venous pressure, and is should be avoided in anterior uveitis. more common ifthe port-wine stain It should also be avoided in aphakic involves the upper lid.Recurrent and pseudophakic patients due to epilepsyand mental retardation the risk of cystoid macular oedema. occur in about 50% of cases. Iris hyperpigmentation results from Cerebral angiomatosis causes increased production of melanin by cerebral calcification in a tram-line the melanocytes.Non-steroidal fashion.Diffuse choroidal anti-inflammatorydrugs which haemangioma is common and inhibit the formation of increases the risk of expulsive prostaglandins have no effect on haemorrhage during intraocular latanoprost,which is a prostaglandin surgery. analogue. 1 The pairing of the following are 1 In Tolosa-Hunt's syndrome: correct: the lesion is located at the orbital epiblepharon - punctate keratitis apex

the cause is usually a metastatic dermatochalasis - superior visual tumour field defect visual loss is common blepharochalasis - allergic dermatitis

orbital inflammation is often absent floppyeyelid syndrome - chronic papillaryconjunctivitis systemic steroids usually relieve the brow ptosis - facial nerve palsy. pain before ophthalmoplegia.

1 Regarding systemic lupus 1 A patient with an orbital implant erythematosus (SLE): develops a superior sulcus deformity. The following methods are useful in the diagnosis is made bythe correcting or reducing the deformity: presence of characteristic anti• nuclear antibodies levator resection the majorityof patients are female of child-bearing age increasing the sizeof the orbital implant superficial punctate keratitis is the most common cornea pathology lower lid tightening associated with SLE

cotton wool spots are a feature of placement of cartilage along the SLE retinopathy orbital floor cerebrovascular accident is the main mucous membrane graft. cause of death. 1 175

In epiblepharon there is excess Tolosa-Hunt's syndrome involves the pre-tarsal skin and orbicularis oculi. cavernous sinus. It is thought to be This can cause misdirection of the an extra-orbitalidiopathic eyelashes,resulting in corneal inflammatory disorder.There is pain irritation and punctate keratitis. and ophthalmoplegia due to Dermatochalasis is seen in the involvement of the third, fourth and elderlydue to excess upper lid skin. sixth cranial nerves. It is a diagnosis Overhanging of the skin can cause of exclusion. As the optic nerve does superior visual field defects. not enter the cavernous sinus, visual Blepharochalasis affects younger involvement is rare. Proptosis is people with recurrent inflammation minimal and orbital inflammation is and swelling of the upper lid.This often absent.Systemic steroid is the can lead to loosening of the skin and treatment of choice. Pain usually levator dehiscence. It is not caused responds to treatment first before byallergy. Floppy eyelid syndrome ophthalmoplegia. typicallyaffects middle-aged overweight males. The upper lid everts easilyduring sleep, causing mechanical irritation.Chronic 176 papillaryconjunctivitis and punctate ~.'.·•..•.·•. b.· •.••·· .•..•• •.•.•.· .••. · •.••. ·• ~.;.;.·.b.·.·.·..·.:;. ~.jbl.·..'·.•.•..·.· •. ,.· ..••. : keratitis are common. Brow ptosis ~~~ can be involutional or the result of Systemic lupus erythematosus is a facial nerve palsy. systemic collagen vascular disease. The underlying pathogenesis is caused byautoimmune necrotising 1 vasculitis. The majorityof patients are female and of child-bearing age. Diagnosis is made clinically bythe Loss of volume or shifting of the presence of specific signs.Superficial orbital implant usually causes a punctate keratitis is the most superior sulcus deformityin an eye common sign in the cornea. Cotton with an orbital implant.Increasing the wool spots are characteristic of SLE sizeof the orbital implant is useful. retinopathy.Proliferative retinopathy the lower lid is lax,tightening the If can occur but is uncommon. The lid will allow the prosthesis to be main cause of death is renal failure. placed more superiorly and reduces the deformity.Placing cartilage in the orbital floor will help to elevate the implant and reduces the deformity. 1 The following B scan ultrasound 1 The pairings of the following features favour a diagnosis of concerning glaucoma are true: choroidal detachment rather than retinal detachment: Scheie's procedure - removal of full-thickness sclera dome-shaped elevation trabeculectomy - partial removal of the ciliary body presence of fluid level goniotomy - incision of trabecular meshwork attachment of vortex veins to the elevated area trabeculotomy - partial removal of trabecular meshwork attachment of the elevation to the optic disc trabeculodialysis - creation of a cyclodialysiscleft. vitreous opacities.

1 The following genetic disorders are associated with the development of multiple basal cell carcinoma:

Corlin's syndrome

xeroderma pigmentosa

incontinentia pigmenti

albinism

neurofibromatosis. 1 1

Choroidal detachment tends to have Scheie's procedure creates a a smooth and dome-shaped outline. full-thickness sclerotomy.Hence, Fluid level is common and the vortex intraocular pressure is less controlled vein may be seen attached to the and hypotonyis common. elevation. Trabeculectomy involves removing part of the sclera to create a sclerotomy which is then covered bya scleral flap.The procedure is performed anterior to the ciliary 1 body.Coniotomy is performed in congenital glaucoma to create a Corlin's syndrome is also called the communication between the naevoid basal cell carcinoma anterior chamber and Schlemm's syndrome.It is an autosomal canal. It involves incision of the dominant disorder characterised by trabecular meshwork at a point development of multiple basal cell between Schwalbe's line and the carcinoma in earlylife. Xeroderma scleral spur. Trabeculotomy pigmentosa is an autosomal establishes a communication recessive disorder.The DNA repair between the anterior chamber and mechanism is defective, resulting in Schlemm's canal by an increased incidence of skin partial removal of the trabecular tumours upon exposure to ultraviolet meshwork (goniotomyab externo). light.Albinism lacks melanin which is Trabeculodialysis involves incision of protective against ultraviolet light.As Schwalbe's line followed by a result, the skin is at risk of various disinsertion of the trabecular tumours including basal cell meshwork from the scleral spur. carcinoma. Incontinentia pigmenti is an X-linked dominant disorder.The patient develops erythema and bullae on the skin and proliferative retinopathy.The risk of developing basal cell carcinoma is not increased. Neurofibromatosis does not cause an increased incidence of basal cell carcinoma. 1 Features of primary congenital 1 The following are true concerning glaucoma include the following: optic nerve trauma in head injuries:

males are more commonly affected it usually involves the intracranial than females portion of the optic nerve

siblings have a 5% chance of being an altitudinal visual field defect is affected common

there is enlargement of both the optic atrophy is a common cornea and the axial length complication

Haab's striae is caused bytears in delaylatency in the visual evoked Descemet's membrane potential may occur

the amount of optic disc cupping it usually responds to optic nerve can be used to predict visual fenestration. prognosis.

1 With regard to AC/ A 1 The following drugs are known to (accommodative convergence/ cause anterior uveitis: accommodation) ratio:

rifabutin it refers to the amount of accommodative convergence in prism dioptres per unit of streptokinase accommodation

it decreases with age quinidine

it is reduced with phopholine iodide cidofovir

it decreases in exotropia metipranolol.

it is better measured with the heterophoria rather than the gradient method. 180 182

Primary congenital glaucoma affects Optic nerve trauma in head injuries males more often than females. often occurs in the intracanalicular About 60 to 80% of cases have portion.The visual field defect is bilateral involvement. The usually a central scotoma. Optic inheritance is multifactorial; siblings atrophy is common and the visual and offspringhave a 5% chance of evoked potential will show a delayed acquiring the disease.The raised latency.Optic nerve fenestration is intraocular pressure causes global usually performed in patients with enlargement of the eye.Haab's striae optic nerve sheath haematoma - results from tears in Descemet's however, in most traumatic optic membrane. Optic disc cupping neuropathies the optic nerve develops earlybut usually reverses appears normal on CT or MRI scan, with treatment of the intraocular and optic nerve fenestration is of pressure. Corneal opacities and uncertain benefit in such cases. refractive amblyopia usually cause poor vision.

183

1 AC/ A ratio refers to the amount of accommodative convergence Other drugs that have been reported needed in prism dioptre per unit to cause uveitis include oral dioptre of accommodation. This ratio contraceptives, hepatitis Band BCG tends to remain unchanged vaccines, interleukin-3, interleukin-6, throughout life.Phopholine iodide diethylcarbamazine and disodium stimulates accommodation and pamidronate. therefore may reduce the ratio. AC/ A ratio is increased in exotropia. AC/ A ratio can be measured with either the gradient or heterophoria method but the former is more accurate. 1 The female carrier of the following 1 Mooren's ulcer: X-linked conditions may show fundal changes: has a better prognosis if the patient is less than 40 years of age choroideremia tends to affect both eyesin older patients retinoschisis is associated with Wegener's granulomatosis in 20% of cases ocular albinism is usually associated with scleritis

retinitis pigmentosa may be treated with limbal conjunctival excision. Norrie's disease.

1 Advantages of fornix-basedover limbal-based conjunctival flaps 1 Features of cluster headaches during trabeculectomy include: include: better surgical exposure greater predilection in males than females less post-operative wound leakage the pupil is typicallyenlarged during the attack easier wound closure ptosis is a feature

formation of a more diffuse bleb prodromal visual symptoms are typical better long-term intraocular pressure rhinorrhoea is common. control. 1 1

In choroideremia, ocular albinism Mooren's ulcer is a peripheral and retinitis pigmentosa, pigmentary ulcerative keratitis that begins in the changes may be observed in the corneoscleral limbus and progresses peripheral retina of the female centrally and circumferentially. The carrier. Retinoschisis and Norrie's condition is idiopathic and not disease are not associated with associated with scleritis.In the abnormal retinal findings in the elderly,the disease tends to be female carriers. unilateral and associated with good prognosis.In the young,especially in people of African origin,the disease tends to be bilateral and more aggressive. Excision of the limbal 1 conjunctival may be useful in some cases. Other treatment options Cluster headaches occur more include systemic steroids and commonly in males than females. immunosuppression. The pain is severe and confined to within the region of the eye.During the attacks, conjunctival redness, lacrimation and Horner's syndrome can occur. Other features include 181 orbital swelling, rhinorrhea and nasal congestion. Unlike migraine, The advantages of fornix-basedover prodromal visual symptoms are limbal-based conjunctival flaps uncommon. during trabeculectomy include: better surgical exposure, easier wound closure and formation of a more diffuse bleb.Post-operative wound leakage is more common in fornix-basedflaps compared with limbal-based flaps.The long-term intraocular pressure control is equivocal in either approach. 188 The following features on B scan 190 Riley-Day'ssyndrome: suggest a rhegmatogenous retinal detachment: is an autosomal recessive disorder

absence of echogenicity behind the membrane causes corneal anaesthesia

attachment of the membrane to the optic disc results in a decrease in lacrimation

full mobilityof the membrane causes decreased sweating

high echogenicity of the membrane may be associated with an increased urine HV A (homovanillic acid). attachment of the membrane to multiple sites away from the disc.

191 In marginal keratitis:

189 Iridoschisis: the lesion is typicallyfound where the cornea rests against the eyelid usually involves the inferiorquadrant of the iris there is a clear area between the lesion and the limbus results from a split between the iris stroma and the pigment epithelium cornea scrape usually yields Gram-positive bacteria can be caused bymydriatics lid swab usually yieldsStreptococcus

is associated with glaucoma in 50% of cases systemic tetracycline can be used to prevent recurrence. can cause endothelial decompensation. 1 190 ~...•....•...••.....•.....••.'....•.••..•• '...... •.....••.•...•.... ~ Rhegmatogenous retinal membranes Riley-Day'ssyndrome or familial are not associated with fluid behind dysautonomia is a rare autosomal the membrane. A fullymobile recessive disorder characterised by membrane suggests a posterior an abnormal autonomic nervous vitreous detachment. The membrane system and sensoryfunction. There has the same echogenicity as the is a deficiency of the enzyme retina.Presence of multiple sites of dopamine ;3-hydroxylase.This causes attachment is suggestive of an increased urinary HV A tractional retinal detachment. (homovanillic acid).The ophthalmic features include corneal anaesthesia, decreased lacrimation, anisocoria and ptosis.Systemic features include paroxysmal hypertension,increased 1 sweating and emotional lability.

Iridoschisis is a degenerative disorder involving the lower quadrant of the iris near the pupil.It is an uncommon 191 condition and usually affects the t$\..·..8...... ·... ·.·...·....· ... ;. ~.·.•• .. ti .•.·.•. ·.·.•. ·.j: ~.e.·..•.•.•.· .•· .•..j elderly.The splittingof the iris occurs VJ:iJ W V!!fjI between the anterior and the Marginal keratitis is an immune posterior stroma. It is idiopathic in disorder caused byhypersensitivity most cases but acute angle-closure to the staphylococcal toxin. glaucoma and use of miotics such as Blepharitis is common. The lesion is pilocarpine are contributing factors. characterised byinfiltrate with a It is associated with glaucoma in clear zonebetween the lesion and 50% of cases and may lead to the limbus. The lesion is usually endothelial decompensation. localised to the area of the cornea which is in close contact with the eyelid.Lid swabs usually reveal Staphylococcusaureus but the cornea scrap is sterile.Treatment consists of weak steroids.Lid hygiene and tetracycline can rid the lids of Staphylococcusand prevent recurrence. 192 In a patient with suspected fungal 194 Posterior polar cataract: keratitis,the following stains can be used for identification: is usually a congenital disorder

haematoxylin and eosin (H&E) involves the cortex and the capsule

periodic acid schiff (PAS) causes more visual symptoms than an anterior polar cataract methenamine silver is associated with a higher incidence of posterior capsule rupture during calcofluor white cataract surgery

is associated with phacodonesis. Giemsa stain.

195 In aqueous tear deficiency: 193 In a pleomorphic adenoma of the lacrimal gland: there is a reduction in the tear meniscus the globe is usually displaced inferomedially facial nerve palsymay be a cause

the tumour contains ductules of the epithelial cells and fattytissue inferior punctate keratopathy is a feature pain is uncommon antihistamine reduces aqueous tear production CT scan usually reveals an increase in orbital volume tear break up time is decreased.

biopsyof the lesion is recommended to exclude malignancy. 1 1

The cell wall of the fungus can be Posterior polar cataracts involve the identifiedusing PAS, methenamine cortex and the capsule. It is a silver, calcofluor white and Ciemsa congenital disorder and can be stain.Calcofluor white also stains sporadic or familial.Visual symptoms acanthamoeba and Ciemsa also are usually more common than stains chlamydia. anterior polar cataracts because of their location and also their tendency to progress. Posterior capsule rupture is a common complication during cataract 1 surgery.Zonular dialysisis not a feature. Pleomorphic adenoma is a benign tumour of the lacrimal gland composed of ductules of the epithelial cells and fibrous myxoid stroma. It grows slowly,causing 1 downward and medial displacement of the globe.Orbital bone erosion In aqueous tear deficiency,the tear without destruction is common, meniscus is reduced and there may resulting in expansion of the orbital be punctate keratopathy of the volume. Pain is rare unless the inferior paracentral region.Facial tumour undergoes malignant nerve palsyaffects the transformation.The treatment of parasympathetic supply to the choice is excision without biopsy lacrimal gland and reduces tear because of the risk of orbital seeding production. Antihistamines, oral leading to recurrence and, rarely, contraceptives and atropine can malignant transformation. cause dryeyes by reducing aqueous tear production. The tear break up time is usually normal in aqueous tear deficiency. 196 In pellucid marginal degeneration: 198 During retinal detachment, the following conditions favour the use the peripheral inferior cornea is of an encircling scleral buckle: thinned and flattened posterior breaks with-the-rule astigmatism is common

U-shaped tear with 'fish-mouthing' Munson's sign is a feature

extensive retinal detachment without Fleischer's ring is commonly seelll detectable breaks

aphakic retinal detachment wedge resection can improve vision.

lattice degeneration in three or more quadrants.

197 In the Herpetic Eye Disease Study:

oral acyclovir is effective in 199 In Brown's syndrome,the following preventing recurrence of herpetic features are observed: keratitis hypotropia of the affected eye oral acyclovir is not effective in preventing uveitis poor elevation in abduction oral acyclovir is effective in preventing stromal keratitis V pattern topical acyclovir is effective in preventing stromal keratitis downshoot in adduction topical corticosteroids are safe and effective in stromal keratitis. widening of the palpebral fissure in adduction. 1 198

Pellucid marginal degeneration is an A radial scleral buckle is preferable in ectatic condition characterised by patients with posterior breaks or thinning and flatteningof the large U-shaped tears to prevent peripheral inferior cornea. This gives 'fish-mouthing'. Other indications for rise to against-the-rule astigmatism. encirclage include: multiple breaks in Munson's sign is a non-specific sign three or more quadrants, mild and can occur in anycondition proliferative vitreoretinopathy, which causes protrusion of the excessive drainage or subretinal fluid. cornea. Unlike keratoconus, Fleischer's ring and Vogt's striae are not seen.Wedge resection of the thinned area can improve vision. 199

Brown's syndrome is characterised bydefective elevation in adduction. 1 The affected eyemay be hypotropic in the primary position.Other Oral acyclovir is effective in features include: V pattern, preventing the recurrence of downshoot in adduction, positive herpetic keratitis and uveitis but forced duction test and widening not stromal keratitis.Topical of the palpebral fissure corticosteroids are safe and effective in adduction. in stromal keratitis. A 40-year-oldpatient presents with a In cystoid macular oedema following 4-week history of persistent right red cataract surgery: eyedespite using chloramphenicol the cause is an increased permeability eyedrops for the past 3 weeks. ofthe perifoveal capillaries Further examination may reveal the following: the majorityof patients are asymptomatic history of facial trauma

a common presentation is poor vision 1 week after cataract extraction delayedfluorescence disappearance test chronic cystoid macular oedema does not respond to treatment history of non-specific urethritis

macular grid laser is effective. positive conjunctival swab for Haemophilus species acne rosacea. The findings of the United Kingdom Prospective Diabetes Study (UKPDS) include:

an improved glucose control reduces The following are true about candida the risk of developing retinopathy in endophthalmitis: typeII diabetes

intensive glucose control with either it occurs exclusively in insulin or oral hypoglycaemic drugs immunocompromised patients can reduce microvascular retinal infiltratesmay give rise to complications Roth spots a decrease in the glycosylated haemoglobin concentration reduces a 'string of pearls' appearance occurs the risk of microvascular with retinal infiltrates complications intravenous fluconazole is the tight control of hypertension reduces preferred treatment rather than microvascular complications amphotericin B an angiotensin-converting enzyme intravitreal amphotericin B is more (ACE) inhibitor is better than a effective than intravenous j3-blocker in preventing amphotericin B. macrovascular complications. This patient has a chronic Cystoidmacular oedema (CMO) conjunctivitis defined as the presence following cataract surgery is caused of conjunctivitis for more than byincreased perifoveal capillary 1 week despite treatment. Blockage permeability.Fluorescein of the nasolacrimal duct can cause angiography shows the condition to chronic conjunctivitis. Therefore, a be common but clinically most history of facial trauma and delayed patients are asymptomatic. CMO fluorescence disappearance test are typicallyoccurs 2-6 months after relevant.Non-specific urethritis may cataract surgery.It is more common be caused bychlamydia which in in the presence of complications turn is a cause of chronic follicular such as a ruptured posterior capsule, conjunctivitis. Acne rosacea is lens malposition and chronic uveitis. Topical adrenaline and latanoprost associated with posterior blepharitis also increase the incidence. which may lead to chronic In patients with chronic CMO conjunctivitis. Haemophilus species associated with lens malposition or can cause acute bacterial adherence of vitreous to the wound, conjunctivitis which responds to lens repositioning or exchange and chloramphenicol. vitrectomy can result in resolution of the oedema. Macular grid laser has not been shown to be effective.

Candida endophthalmitis occurs in disseminated candidiasis and can The UKPDS is a randomised, occur in immunocompetent patients prospective clinical trial of typeII such as intravenous drug users. The diabetes.The study shows that lesion usually begins in the subretinal intensive control of blood glucose areas and extends into the retina and with either insulin or oral the vitreous. Roth spots can occur hypoglycaemic drugs slows down with retinal infiltrates.Vitreous the progression of retinopathy and infiltratecan give rise to a 'string of other microvascular complications of pearls' appearance. Amphotericin B diabetes.Intensive blood pressure is highly protein bound and therefore control slows down the progression penetrates ocular tissue poorly. of retinopathy and reduces the risk Systemic fluconazole is the of both microvascular and treatment of choice for ocular macrovascular complications of endophthalmitis. In severe cases, diabetes.There are no clinical vitrectomy and intravitreal differences between ACE inhibitors amphotericin B are effective. and ,B-blockers 204 AMPPE (acute multifocal placoid 206 Disc drusen: pigment epitheliopathyl: are of similar histologyto retinal is usually a bilateral condition drusen

become progressivelyembedded affects young males more commonly with age than females are associated with peripapillary is a self-limitingcondition haemorrhage

are associated with angioid streaks shows hypofluorescence in the early phase of fluorescein angiography are associated with retinitis shows reduced ERG and EOG pigmentosa. recordings.

207 Plus disease in retinopathy or 205 Pavingstone retinal degeneration: prematurity is characterised by:

is present in 50% of people over the vitreous haze age of 20

represents areas of ischaemic outer lens opacities retina

is found mainly in the superior pupillary rigidity quadrants

does not predispose to retinal breaks corneal haze

coalesces and gives rise to tortuosityof retinal vessels. retinoschisis. 206

First described byCass in 1968,acute Disc drusen are of different posterior multifocal placoid pigment composition to retinal drusen. With epitheliopathy (APMPPE) is an age, the drusen become exposed. acquired inflammatory disorder They are associated with affecting the retina, retinal pigment peripapillarysubretinal neovascular epithelium and choroid of otherwise membrane formation, angioid young healthy adults, with equal streaks and retinitis pigmentosa. male to female incidence. The disease is self-limitedand is characterised bymultiple yellowish-white placoid subretinal lesions of the posterior pole.The 207 lesions are frequentlybilateral and in various stages of evolution, typically Plus disease implies dilation and resolving in weeks to months and tortuosityof the blood vessels near leaving circumscribed areas of retinal the optic nerve. It also includes the pigment epithelial disturbance. growth and dilation of abnormal Fluorescein angiography shows blood vessels on the surface of the hypofluorescence in the earlyphase iris,rigidity of the pupil and vitreous and this is either the result of haze.The diagnosis of plus disease is blockage bythe RPE or choroidal usually based on the appearance of ischaemia. Electrophysiologytests the vessels near the optic nerve, as show reduced ERG and EOG compared with standard retinal recordings. photographs. The presence of plus disease suggests a more fulminant or rapidlyprogressive course .

•.....•.•...... •....•...•..•....•...•..•...... ~ •'-';i;;~:::-h' Pavingstone retinal degeneration represents discrete areas of ischaemia involving the outer retina. Itis seen in 22% of patients over the age of 20.It is most commonly located in the inferior retinal quadrant anterior to the equator. Itdoes not give rise to retinal breaks. Retinoschisis is caused by coalescence of peripheral cystoid degeneration. Common ocular findings in o In vernal keratoconjunctivitis: Goldenhar's syndrome include: the condition is more common in hypertelorism cold than warm climates

cobblestone appearance of the eyelidcoloboma conjunctiva is seen almost exclusively in the upper tarsal conjunctiva optic disc coloboma interpalpebral corneal staining is a feature microphthalmos a shield ulcer seen in the superior cornea is always surrounded by decreased corneal sensation. infiltrates

eyerubbing increases the severity of symptoms.

The following features distinguish optic disc swelling from papilloedema: 1 The following are indications for temporal clear corneal incision under the presence of relative afferent local anaesthesia: pupillarydefect an unco-operative patient normal visual acuity

deep sulcus absence of venous pulsation

presence of a superior filteringbleb leakage of fluorescein from the optic nerve head during fluorescein angiography with-the-rule astigmatism an enlarged blind spot. small pupil. o

Goldenhar's syndrome is also called Vernal keratoconjunctivitis is more oculoauriculovertebral dysplasia. common in warm than cold Eye signs include Iimbal dermoid, lid climates. It is characterised bythe coloboma and microphthalmos. presence of conjunctival cobblestones confined almost exclusively to the superior tarsal conjunctiva. Interpalebral corneal staining is a feature of dryeye. Shield ulcers are not associated with corneal infiltration.Eye rubbing can Optic disc swelling causes abnormal increase the discharge of mast cell optic nerve function with a relative mediators and increase the severity afferentpupillary defect, colour of symptoms. desaturation and decreased visual acuity.Absence of venous pulsation, an enlarged blind spot and leakage of fluorescein from the optic nerve head occur in both optic disc 1 swelling and papilloedema. The advantages of a temporal clear corneal incision include less astigmatic changes because the horizontal corneal diameter is longer than the vertical diameter. In addition,it allows easyaccess to the eyein patients with sunken globes.A superior filteringbleb may be compromised ifthe phacoemulsification is performed through a superior corneal incision. An unco-operative patient should have general anaesthesia. A temporal incision will worsen with-the-rule astigmatism. The location of the incision site does not affect pupil size. 2 With regard to cataract surgery in a 4 Meningioma of the orbit: patient with diabetes mellitus: results from intracranial extension in progression of non-proliferative 90% of cases diabetic retinopathy in the operated eyeis a feature occurs more commonly in females than males progression of non-proliferative diabetic retinopathy in the non• is associated with neurofibromatosis operated eyeis a feature typeI

progression of non-proliferative is more oftenbenign in children than diabetic retinopathy is more adults common in women than men can be treated with chemotherapy. a small capsulorrhexis is preferable to avoid extrusion of the implant

an implant with a large optic is preferable. 5 In a 6-month-old child with poor vision, myopic refraction is seen in:

gyrateatrophy 3 The following techniques are useful in repairing a large lower lid defect involving the margin following Leber's congenital amaurosis excision of basal cell carcinoma:

Cutler-Beard's procedure X-linked congenital stationary blindness

Tenzel flap blue cone monochromatism

Hughes' procedure Norrie's disease.

Mustarde rotation flap

glabellar flap. 2 214 ~.••••••. '•••••• '.•• '.' •••••• ' .•••••• C•••• '•.'.' ••' •• ' .•.•..•••••••.••.•••• ~ Diabetes mellitus is associated with Only 10% of orbital meningiomas progression of non-proliferative arise within the orbit,the rest result diabetic retinopathy in the operated from intracranial extension,usually eyefollowing cataract surgery.This from the sphenoid bone or the progression is more common in olfactorygroove. It is more common women, in particular those who are in females than males and is overweight. A large capsulorrhexis associated with neurofibromatosis and an implant with a large optic are typeI. In children, orbital preferable as this would allow better meningioma tends to be more visualisation of the periphery and aggressive with a propensity facilitate pan-photocoagulation. for intracranial spread. Well-circumscribed meningioma can be excised and the diffuse form treated with radiotherapy. Chemotherapy is not useful.

3 ~..•...... '.•.~.,' ..'...•.' , •..•..•...... '.; •.•.•...... '•.. ~ Cutler-Beard's procedure is used in the repair of upper lid defect. A glabellar flap is used for the repair 215 of medial canthal skin defects. Gyrate atrophy is associated with myopia but the vision is not affected until adulthood. Leber's congenital amaurosis is associated with . hypermetropic refraction.Myopic shift is a feature of X-linked congenital stationaryblindness and blue cone monochromatism. 6 Features of de Morsier's syndrome 8 Diabetic papillopathy: include: results from intracranial hypertension optic nerve hypoplasia

is a feature of poor diabetic control nystagmus

is associated with optic disc growth failure neovascularisation

usually resolves with laser absent septum callosum photocoagulation

indicates poor visual prognosis. basal encephalocele.

9 In superior oblique myokymia: 1 The effects of pregnancy include: there is repetitive large oscillation of enlargement of uveal melanoma the affected eye

ocular oscillation can be heard with progression of Grave's disease a stethoscope

the oscillation is usually precipitated reduction in the sizeof prolactinoma byreading

there is an association with increased incidence of diabetic cerebellar lesion macular oedema carbamazepine is useful. enlargement of meningioma. 6 1!!J0\...... '...... ,.,....,... ,.,.'.'...... ,. '...••.... '...... •..,....•. . .. 218 None ~ de Morsier's syndrome or septo-optic Diabetic papillopathyis an uncommon complication of diabetes dysplasiaconsists of a triad of optic mellitus. It can occur in young or old nerve hypoplasia,nystagmus and patients and in typeI or typeII growth failure.Neuro-imaging diabetes.There is no intracranial typicallyshows an absent septum hypertension,the swelling may be pellucidum and basal encephalocele. unilateral or bilateral.Poor diabetic control is not a feature and laser photocoagulation does not help. The swelling may be the result of optic 7 nerve head ischaemia, and telangiectasia may be seen, which During pregnancy, uveal melanoma, can be confused with prolactinoma and meningioma can neovascularisation. The swelling undergo enlargement. Grave's usually resolves spontaneously and disease becomes more severe. the vision is not affected. Diabetic retinopathy of anytype tends to progress during pregnancy.

9

Superior oblique myokymia is a rare disorder characterised byrepetitive small amplitude oscillation of the globe.This oscillation may be vertical or oblique. The patient usually complains of diplopia,blurred vision or oscillopsia.The oscillation is best observed with a slit-lamp and can be heard with a stethoscope. It is often precipitated byreading. The condition is benign and a central nervous disorder is uncommon. Carbamazepine can reduce the oscillation. The pairing of the following are Proliferative vitreoretinopathy: correct: results from proliferationof retinal opsoclonus - pituitarylesion pigment epithelial cells

is more common in younger patients ocular bobbing - coma

is increased in the presence of ocular flutter - cerebellar lesion vitreous haemorrhage

is more common in retinal ocular myoclonus - pontine stroke detachment treated with a scleral buckle compared with a vitrectomy spasmus nutan - neuroblastoma. can be prevented with systemic steroid.

In a patient with a choroidal Features of multiple evanescent melanoma, the risk of metastasis is white dot syndrome include the increased in the presence of: following: epithelioid cell type female predilection

an anterior location presence of an enlarged blind spot despite a normal optic disc involvement of the vortex vein vitritis being a prominent feature

the tumour extending through the fluorescein angiography shows early Bruch's membrane hyperfluorescence of the white lesions a large tumour. electrophysiologystudies indicate dysfunction of the inner retinal layer. 220 222

Opsoclonus is characterised by Proliferative vitreoretinopathy is a spontaneous chaotic rapid conjugate late complication of retinal surgery. bursts of ocular saccades in all It is caused byproliferation of retinal directions.In children it may be pigment epithelial cells on the retina. associated with neuroblastoma. It is associated with vitreous Other associations include occult haemorrhage and vitreous surgery. visceral carcinoma in adults and Conditions that release retinal post-viral encephalitis. Ocular pigment epithelial cells into the bobbing is characterised byfast vitreous increase the risk of downward ocular movements proliferative vitreoretinopathy,and followed bya slow return ofthe eyes include multiple or large breaks and to the primary position.It is seen in the use of cryopexy. comatose patients with major intracranial haemorrhage. Ocular flutter involves to-and-frohorizontal oscillations that occur when the patient attempts to fixateon a: target 223 in the primary position.It is associated with cerebellar lesion. Multiple evanescent white dot Ocular myoclonus initiallyresembles syndrome is often a unilateral ocular bobbing and then becomes condition affecting young female more pendular. It is seen in patients patients.The presentation is usually with pontine stroke. Spasmus nutan decreased vision with multiple white is made up of a triad of nystagmus, dots in the retinal pigment involuntary head movement and an epithelium layer.An enlarged blind abnormal head posture. It usually spot is common despite a normal resolves spontaneously and has no optic disc. Vitritis is usually mild. underlying cause. Fluorescein angiography shows early hyperfluorescence of the white 221 lesions.ERG shows a decreased a-wave, suggesting abnormal outer The risk of metastasis in choroidal retinal dysfunction.The condition melanoma is increased in the usually resolves spontaneously with presence of a large tumour, an good visual recovery. anterior location of the tumour, a tumour composed mainly of epithelioid cells and involvement of the vortex vein. Extension of the tumour through Bruch's membrane is not associated with metastasis. In lamellar cataracts: Periorbital haematoma is a feature of: the lens opacity is between the cortex and the embryonic nucleus basal skull fracture

progression of the cataract after birth is common amyloidosis

there are usually linear opacities extending from the cataract into the neuroblastoma clear peripheral cortex the insult leading to the cataract haemophilia A usually occurs in the first trimester

abnormal maternal calcium sickle cell anaemia. metabolism is a cause.

The following are true about In a patient whose pupils do not piggyback intraocular lens implant: react to light but constrict with accommodation, the following signs it is usually carried out in patients may be an associated finding: with a refraction of more than 30D vermiform movement of the eye spherical aberrations are more common than using one single lens small irregular pupils a larger incision is required during surgery presence of optic atrophy in both the lens with higher power is usually eyes placed anterior to the lens with lower power presence of extensive pan-photocoagulation interlenticular opacification is a complication. convergence-retraction nystagmus. '\i:l!JJ~.•.e.>,n Lamellar or zonular cataract is a Basal skull fracture can cause congenital cataract characterised by bilateral periorbital haematoma the presence of lens opacities (racoon's sign).Amyloidosis may lead between the cortex and the to formation of fragile blood vessels embryonic nucleus. Two additional which may bleed spontaneously. features are the presence of an Orbital metastasis of neuroblastoma envelope, which is a thin opacity is also associated with spontaneous surrounding the main cataract, and haemorrhage. Haemophilia A results the presence of riders, which are in an abnormal clotting profile but linear opacities extending from the does not cause spontaneous cataract into the clear cortex. The bleeding and neither does sickle insult is thought to occur in late cell anaemia. pregnancy.Abnormal maternal calcium metabolism such as hypoparathyroidism is a known factor.The condition is usually static. •.•.,.•. a .•. ·.•.•. ·.·.•. ·.•.•.•.. · .•·.•. ... C3.•.•.'.•• '.•.•. '.•. '.".,.'. '

Anton's syndrome - bilateral parietal lobe lesion. Leber's congenital amaurosis.

In retinitis pigmentosa: 1 Kaposi's sarcoma: the X-linked typetends to have worse visual prognosis than the is caused byhuman herpes virus autosomal dominant type type8 electrophysiologytypically shows arises from the endothelium a normal a-wave but abnormal b• wave typicallycauses blockage of visual field shows ring scotoma best lymphatic drainage shown on Humphrey perimetry is confined to the skin the macula may have oedema which does not leak on fluorescein angiography can be treated with antiviral agents. the sectorial typetypically involves the inferior retina. 230

Gerstmann's syndrome results from a Other conditions associated with lesion in the dominant parietal lobe. deafness include: It is characterised byacalculia, • Usher's syndrome agraphia, finger agnosia and left-right • mucopolysaccharidoses confusion. Prosopagnosia is an • Alport's disease inabilityto recognise familiar faces • congenital syphilis and usually results from bilateral • congenital rubella occipitotemporal involvement. • Vogt-Koyanagi-Harada's Achromatopsia is abnormal colour syndrome. discrimination and can arise from bilateral parietal or occipital lobe abnormalities.Akinetopsia implies insensitivityto motion and can result from a lesion in the middle temporal 231 gyrus.Anton's syndrome occurs in blind patients who denythey are Kaposi's sarcoma is most commonly blind.It is seen in bilateral occipital seen in AIDS patients.It is caused lesions. byhuman herpes virus type8. The tumour originates in the endothelium of either blood or lymphatic vessels.The tumour commonly invades the lymphatic system,causing lymphoedema. Retinitis pigmentosa may be It can affect not onlythe skin but autosomal dominant, X-linked or the conjunctiva and orbit.Antiviral autosomal recessive. The autosomal agents do not appear to be effective. dominant typetends to have the Excision, radiation and systemic best visual prognosis.ERG shows chemotherapy with either vincristine abnormal a- and b-waves with or vinblastine are effective. eventual obliteration of both. The ring scotoma involves the periphery and is best shown on Goldman visual field.Cystoid macular oedema may occur but fluorescein angiography may not reveal leakage. Oral acetazolamide may be helpful. Sectorial retinitis pigmentosa is a variant and usually involves the lower retina. The cover test: During repair of a rhegmatogenous retinal detachment, the following are gives the same information as the true regarding drainage of subretinal alternating cover test fluid: can detect abnormal retinal drainage is indicated if there is correspondence significant vitreoretinal traction can detect eccentric fixation drainage should be performed as close to the retinal hole as possible is the best test for tropia it can help to localise the retinal hole

can be used to detect dissociated it is useful for softening the eye vertical deviation.

drainage should be performed before buckling.

In pituitaryapoplexy: severe sudden headache is the typical presentation Regarding herpes virus infections: decreased vision with conjunctival swapping for Lipschutz ophthalmoplegia is common bodies is found in herpes simplex viral infections occurs mostly in malignant tumour Giemsa staining of epithelial cells helps in the diagnosis of herpes urgent resection of the tumour with simplex viral infections steroid cover is necessary fluorescent antibodystaining hypopituitarism is common. techniques demonstrate intracellular herpetic inclusion bodies

varicella zosterinfection of the lids can be confirmed bystaining for Henderson Patterson bodies

Tzanck smear is useful for identifying herpes virus. 234

The cover test is used to detect The indications for drainage of tropia,whereas the alternating subretinal fluid are: presence of a cover/uncover test detects both bullous retinal detachment which is tropia and phoria. Abnormal retinal making localisation of a retinal tear correspondence and eccentric or hole difficult;long-standing retinal fixationcannot be detected with a detachment with viscous subretinal cover test.Dissociated vertical fluid;and inferior retinal detachment. deviation shows an upward Drainage should be performed movement of the eyewhen it is before buckling and away from the covered. retina break, as the use of cryotherapy at the site of drainage may cause problems.

235

Pituitary apoplexyoccurs as a result Lipschutz bodies,also called Cowdry of sudden enlargement of a pituitary typeA inclusions, are seen in cells adenoma either due to haemorrhage infected with herpes simplex or or infarction.The tumour is usually varicella zostervirus. They are best benign.The resultant mass can seen with the infected epithelial cells expand into the cavernous sinus, fixed in Bouin's solution and stained causing compression of the cranial with Papanicolaou method. Giemsa nerves. Altered consciousness with stain shows up cells infected bythe decreased vision and herpetic virus. The infected cells ophthalmoplegia are common. In the show multi-nucleation with balloon acute stage, resection of the tumour degeneration.Immunofluorescence with steroid cover can be life-saving. can identifycells infected with Hypopituitarism is a common herpes virus. Henderson Patterson outcome with or without surgery. bodies refer to eosinophilic cytoplasmic inclusion bodies seen in molluscum contagiosum. Tzanck smear involves scraping the base of a vesicle and inoculating it onto a microscopic slide.The syncytial giant cells which are characteristic of cells infected with the herpes virus can be demonstrated with Wright's or Giemsa stain. Superior limbic keratoconjunctivitis: Enlargement of the optic canal occurs in: is associated with superior follicular tarsal conjunctivitis carotid cavernous fistula causes hypertrophy of the superior limbus fibrous dysplasia is associated with filamentary keratitis sphenoidal meningioma can be diagnosed with impression cytology ophthalmic arteryaneurysm shows a favourable response with silver nitrate to the upper tarsal optic nerve glioma. conjunctiva.

The ocular complications of bone Vitamin B12 deficiency: marrow transplantation include: produces a decrease in central vision cataract with relative sparing of peripheral vIsion keratoconjunctivitis sicca does not affect colour vision

cotton wool spots may cause retinal haemorrhages

cicatricial lagophthalmos causes temporal pallor of the optic nerve in long-standing cases corneal keratinisation. is the actual cause of alcohol• tobacco amblyopia. 236 238

Superior limbic keratoconjunctivitis A lesion ofthe optic nerve can cause is a chronic condition involving the enlargement of the optic canal and superior limbus and cornea and the this is seen in optic nerve superior tarsal conjunctiva. The meningioma and glioma. changes in the tarsal conjunctiva are Ophthalmic arteryaneurysm also typicallypapillary. The limbus is causes enlargement. Carotid often thickened. Although the cavernous fistula does not affect the diagnosis is usually made clinically,in canal size.Sphenoidal meningioma doubtful cases, impression cytology causes hyperostosis and may reduce of the superior bulbar conjunctiva the sizeof the canal. can be useful and this usually shows nuclear pyknosis,loss of goblet cells and keratinisation.Treatment includes the use of silver nitrate to 239 the upper tarsal conjunctiva, ~.•• ,..•.·.; ..••.•. a.·.· •.·.••.·.•.•.· •.•.· •.; .•• •.•.·..•... ;.;...... ; ' 1fJJA.·'•.·.••. ·•.••. ··;: •.G.· ..;.·.;.;.••• ••••.••• ·.•.••.• •. ·.• .•.••. i..·; resection of the bulbar conjunctiva ~ 'fif!!ffJ and the use of a large-diameter Vitamin B12 deficiency may give rise contact lens. to optic neuropathy. Colour vision is affected and the visual field may show central or centro cecal scotoma. The posterior pole typically shows flame-shaped and dot-blot retinal haemorrhages. Temporal pallor of the optic nerve is common. Following bone marrow It is a cause of amblyopia in transplantation some patients can alcohol-tobacco amblyopia. develop graft-versus-host disease. In the chronic form, patients may develop cataract, ocular surface abnormalities such as keratoconjunctivitis sicca, corneal keratinisation and skin abnormalities such as cicatricial lagophthalmos. Rarely,cotton wool spots and haemorrhages have been observed. The following lesions may be responsible for a Horner's pupil that dilates with 0.1% adrenaline:

Pancoast's tumour

syringomyelia

post-thyroidectomy

internal carotid arteryaneurysm

post-sympathectomy for Raynaud's phenomenon. A normal pupil does not react to 0.1 % adrenaline. However, in patients with post-ganglionic third-order lesions, denervation hypersensitivity develops, causing the pupil to dilate with 0.1 % adrenaline. Pancoast's tumour causes a second-order lesion and syringomyelia causes a first-order lesion. Third-order lesions may occur in post-thyroidectomy patients, internal carotid artery aneurysms and post-sympathectomy. Fact Finder

5-fluorouracil, endophthalmitis 74 adult epiphora 28 aesthenopia 30 a blockers, Sturge-Weber's syndrome 28 age-related macular degeneration (ARM D) 4 A pattern strabismus 87,88 VEP 42 abduction deficit 72 AIDS patients aberrant regeneration, oculomotor nerve CMV retinitis 44 3,4 HAART 38 abetalipoproteinaemia 50,58 Kaposi's sarcoma 120 AC/ A ratio 95,96 PORN 44 acanthoamoeba keratitis 45,46 akinetopsia, middle temporal gyrus lesion accommodative esotropia 79,80 120 acetazolamide albinism 41, 42,94 contraindications 54 alcohol, optic neuropathy 114 glaucoma 40 alcohol-tobacco amblyopia 124 Steven-johnson's syndrome 54 alcoholism 8,10 Sturge-Weber's syndrome 28 alkaline burns 77,78 acetylcholine, cataract surgery 88 Alport's syndrome 16 acetylcholine receptor antibodies 62 deafness 120 achromatopsia 58 aluminium foreign bodies 62 bilateral occipital lobe lesion 120 amblyopia 68 acne rosacea 106 myelinated nerve fibres 34 acoustic neuroma 75,76 vitamin B12 defiCiency 124 neurofibromatosis 60 amiodarone acute multifocal placoid punctate epitheliopathy anterior stellate deposit 16 (AMPPE) 107,108 optic neuropathy 114 vitritis 84 vortex keratopathy 16 acute retinal necrosis 19,20 amniotic membrane transplant,stem cell acyclovir deficiency 48 acute retinal necrosis 20 AMPPE see acute multifocal placoid punctate herpetic keratitis 104 epitheliopathy adenoviral keratoconjunctivitis 47,48 Amsler's sign 74 Adie's pupil 2, 126 amyloid,lattice dystrophy 80 vermiform eye movement 118 amyloidosis,periorbital haematoma 118 adrenaline 2,125 anaesthesia, peri bulbar 30,63, 64 anisocoria 126 anatomical limbus 8 macular oedema 2 ANCA 10 miotic pupil 126 angioid streaks, disc drusen 108 third order lesions 126 angle-closure glaucoma 18,28, 81, 82 aniridia 57, 58 b-wave amplitude reduction 40 zonular dehiscence 18 band keratopathy 60 anisocoria 52 par planitis 86 diagnosis 126 Bardet-Biedl's syndrome, deafness 120 anterior chamber shallowing 17, 18 basal cell carcinoma 93,94,111,112 anterior lenticonus 16 basal encephalocele, de Morsier's syndrome 114 anterior stellate deposit 16 BCG vaccine, uveitis 96 anterior synechiae 28 Bell's palsy, unilateral facial nerve palsy 62 anterior transposition 8 benign intracranial hypertension 67, 68 superior oblique muscle 8 Best's disease 24 anterior uveitis, latanoprost 90 bilateral occipital lobe lesion, achromatopsia 120 antibiotics biometry, intraocular lens calculation cataract surgery 88 84, 85, 86 erythromycin 22 blind spot, multiple evanescent white dot intravenous 18 syndrome 116 prophylactic 32 blink reflex, acoustic neuroma 76 antihistamines 102 blinking, excessive, essential blepharospasm 48 Anton's syndrome 120 blood tests 9, 10 Apert's syndrome 88 blowout fracture 77, 78 aphakic eyes, silicone oil 84 blue cone monochromatism 112 aphakic patients, latanoprost 90 bone marrow transplantation 123, 124 applanation tonometry 55, 56, 85, 86 bone-spicule pigmentation 6, 57, 58 apraclonidine, contraindications 54 botulinum toxin 25, 26, 48 aqueous misdirection, cycloplegic 18 branch retinal vein occlusion 35, 36 aqueous tear deficiency 101, 102 brimonidine, Sturge-Weber's syndrome 28 argon laser trabeculoplasty 27, 28 brow ptosis, facial nerve palsy 91, 92 ARMD see age-related macular degeneration Brown's syndrome 103, 104 Arnold-Chiari malformation 52 Brushfield's spots, Down's syndrome 56 arthritis see juvenile chronic arthritis; juvenile idiopathic arthritis C-ANCA see cytoplasmic ANCA aspirin C-reactive protein, giant cell arteritis 26 carotid artery stenosis 72 calcification, retinoblastoma 44 diabetic retinopathy 34 calcofluor white (stain) asteroid hyalosis 86 acanthoamoeba keratitis 46 astigmatic keratotomy 110 fungal keratitis 102 astigmatism candida endophthalmitis 105, 106 cataract surgery 110 canthaxanthine, crystalline retinopathy 2 limbal dermoids 76 capsulorrhexis 13, 14, 30, 84, 112 atrophic holes 74 capsulotomy 13, 14 atropine 102 carbamazepine, superior oblique myokymia 114 oculocardiac reflex 30 carotid artery stenosis 71, 72 autofluorescence 5, 6 carotid cavernous fistula 22 cataract f3 blockers, Sturge-Weber's syndrome 28 bone marrow transplantation 124 B scan ultrasound features 93, 94, 99, 100 congenital rubella 77, 78 and glaucoma 78 choroidal melanoma 35,36 lamellar 117,118 metastasis 115,116 macular hole surgery 40 choroidal naevus 35,36 perimetry 56 choroidal neovascularisation 72 posterior polar 101,102 choroideremia 98 zonular 117,118 choroidoretinal atrophy 58 cataract surgery 29,30, 73, 74 chronic open angle glaucoma 42 astigmatism 110 chronic papillaryconjunctivitis, floppyeyelid chronic primary angle-closure glaucoma 82 syndrome 91,92 diabetes mellitus 111,112 chronic primary angle-closure glaucoma endophthalmitis 78 81,82 expulsive suprachoroidal haemorrhage 14 chronic progressive external ophthalmoplegia Fuch's heterochromic cyclitis 74 head posture 66 medication 87,88 ptosis surgery 68 posterior capsule rupture 102 cicatricial entropion 88 sympathetic ophthalmia 44 cicatricial lagophthalmos, bone marrow zonular dehiscence 17, 18,42 transplantation 124 cavernous sinus thrombosis 22 cidofovir CD4+ counts 20 CMV retinitis 70 CMV retinitis 44 uveitis 96 CD4 + function, cyclosporin 64 ciliary bodyablation 53,54 central areolar choroidal dystrophy 24 cirrhosis, primary biliary 2 central retinal arteryocclusion 69,70 cloudy cornea see corneal clouding central retinal vein occlusion 35,36 cluster headaches 98 b-wave amplitude reduction 40 CMO see cystoid macular oedema central serous retinopathy 39,40 CMV retinitis see cytomegaloviral (CMV) cerebellar haemangioblastoma, retinal capillary retinitis haemangioma 20 cobblestone appearance conjunctiva 110 cerebellar lesion,ocular flutter 116 cocaine test chalazion, foreign bodygiant cells 62 Horner's syndrome 2 Chediak-Higashi syndrome 42 miotic pupil 126 cherry red spot, ophthalmic arteryocclusion Cogan's interstitialkeratitis, deafness 120 70 coma, ocular bobbing 116 chiasmal region lesions 52 congenital cataract 52 chlamydial conjunctivitis 22,106 congenital esotropia 82 chloramphenicol congenital glaucoma 78 conjunctivitis 105,106 congenital hereditary endothelial dystrophy 78 optic neuropathy 114 congenital nasolacrimal duct obstruction (hydroxylkhloroqine 25,26 retinal pigment epithelial dysfunction 2 congenital nystagmus 59, 60 vortex keratopathy 16 congenital rubella 77,78 chlorpromazine, vortex keratopathy 16 deafness 120 chlorpropamide, optic neuropathy 114 congenital syphilis 2 choristomas 76 deafness 120 choroidal circulation 84 conjunctival erythema 78 conjunctivitis cryopexy,PVR 116 chlamydial 22,106 cryotherapy chronic lOS, 106 cerebellar haemangioblastoma 20 gonococcal 22 familial exudative vitreoretinopathy 22 neonatal 21,22 phaeochromocytoma 20 contact lenses, acanthoamoeba keratitis 46 pneumatic retinopexy 14 contraceptives, oral 96 PVR 16 contraindications, drug 53,54 crystallineretinopathy, drugs causing 2 contralateral inferior rectus recession 8 cutis marmarata telangiectasia congenita 38 convergence retraction nystagmus 52,118 Cutler-Beard's procedure 112 Parinaud's syndrome 118 cycloplegic, aqueous misdirection 18 copper foreign bodies 62 cyclosporin 63,64 corneal anaesthesia, Riley-Day'ssyndrome 100 cycoplegic refraction, accommodative corneal blood staining 23,24 esotropia 80 corneal clouding 49,50, 53, 54 cystoid macular oedema (CMO) 38,lOS, 106 STUMPED mnemonic 54 cytomegaloviral (CMV) retinitis corneal graft AIDS patients 44 cyclosporin 64 diagnosis 19,20 keratoconus 50 HAART 37, 38,44 corneal keratinisation,bone marrow management 69,70 transplantation 124 cytoplasmic ANCA (C-ANCA) 10 corneal neovascularisation, aniridia 58 corneal perforation,ionising radiation 80 D-penicillamine treatment 2 corticosteroid-induced ocular hypertension dacryocystocele,congenital nasolacrimal duct 23,24 obstruction 26 corticosteroids dacryoliths 28 ONTT 6 Dalen-Fuchs' nodules, sympathetic oral 6 ophthalmia 44 see alsosteroids DCR, external/endonasal 28 corticosteroids, systemic 6 de Morsier's syndrome 113,114 orbital myositis 88 deafness 119,120 par planitis 86 delayedfluorescence disappearance test corticosteroids, topical, stromal keratitis lOS, 106 104 dendritic ulcers 65,66 Corynebacterium 10 dentition,abnormal 1, 2 Corynebacterium diphtheriae 76 dermatochalasis, superior visual field defect 91, cosmetically unacceptable enophthalmos, blow 92 out fracture 78 Descemet's membrane 2, 50,96 cotton wool spots 20 desferrioxamine, retinal pigment epithelial bone marrow transplantation 124 dysfunction 2 SLE 92 deuteranomaly viii-ix cover test 121,122 Diabetes Control and Complications Trial Cowdry typeA inclusions, herpes virus (DCCT) 10,57, 58 infections 122 diabetes mellitus 4 Crouzon's disease 88 cataract surgery 111,112 corticosteroid-induced ocular eighth cranial nerve, acoustic neuroma 76 hypertension 24 endarterectomy, carotid artery stenosis 72 glycosylated haemoglobin assay 70 endophthalmitis 105, 106 macular oedema 9, 10 cataract extraction 78 UKPDS 10, 105, 106 cataract surgery 88 vitrectomy surgery 54 Nd:Y AG laser capsulotomy 80 diabetic papillopathy 113, 114 trabeculectomy 73, 74 diabetic retinopathy 10, 33, 34 Endophthalmitis Vitrectomy Study 17, 18 pregnancy 114 endothelial dystrophy, cloudy cornea 54 Diabetic Retinopathy Study (DRS) 37, 38 endothelial proliferation 77, 78 diethylcarbamazine, uveitis 96 enlarged superior ophthalmic vein 21, 22 digitalis, optic neuropathy 114 enophthalmos 52 dilated pupil, fixed 1, 2 blowout fracture 78 diplopia 12, 30 epiblepharon, punctate keratitis 91, 92 blowout fracture 78 epilepsy, Sturge-Weber's syndrome 90 internuclear ophthalmoplegia 64 epiphora, adult 28 vertical/torsional 8 epiphyseal dysplasia 6 dipyridamole, carotid artery stenosis 72 epiretinal membrane 40 disc drusen 107, 108 epithelial down growth 6 disodium pamidronate, uveitis 96 Erdheim-Chester's disease, Touton giant dissociated vertical deviation 12 cells 62 divisional oculomotor nerve palsy 51, 52 erythema multiforme 50 DNA synthesis, mitomycin C 48 erythromycin, neonatal conjunctivitis 22 dominant parietal lobe, Gerstmann's esotropia syndrome 120 congenital 82 down beat nystagmus 52 Down's syndrome 56 down-gaze palsy 4 infantile 11, 12 Down's syndrome 55, 56 essential blepharospasm 47, 48 drainage, subretinal fluid 122 ethambutol, optic neuropathy 114 DRS see Diabetic Retinopathy Study examiner's tricks v-ix drug contraindications 53, 54 exotropia 29, 30 drusen 4 divisional oculomotor nerve palsy 52 disc 107, 108 expulsive suprachoroidal haemorrhage dry eye 13,14 goblet cells 50 eyemovements 3, 4, 51, 52 ionising radiation 80 eyelid defects 112 Steven-johnson's syndrome 50 eyelid malposition, Steven-johnson's dual information, examiner's tricks vi-vii syndrome 50 eccentric staphyloma 85, 86 facial nerve palsy echocardiogram, Marfan's syndrome 16 aqueous tear deficiency 102 ectopia lentis brow ptosis 91, 92 syphilis 16 familial exudative vitreoretinopathy 21, 22 urine thiosulphate 16 fat atrophy, involutional entropion 88 Ehler-Danlos syndrome, zonular dehiscence 18 FFA see fluorescein angiography fixed dilated pupil 1, 2 congenital 78 floppyeyelid syndrome, chronic papillary corticosteroid-induced ocular hypertension conjunctivitis 91,92 24 fluconazole,candida endophthalmitis 106 goniotomy 94 fluorescein angiography (FFA) 83,84 intraocular pressure 42 AMPPE 108 lOP 18 branch retinal vein occlusion 36 iridoschisis 100 multiple evanescent white dot syndrome malignant 17, 18,28 116 mitomycin C 48 sickle cell disease 68 normal tension 55, 56 fluorescein, applanation tonometry 56 phacolytic 28 5-fluorouracil, endophthalmitis 74 phacomorphic 28 f1urbiprofen,cataract surgery 88 pigmentary 4 folatedeficiency 46 pneumatic retinopexy 14 foreign bodies,intraocular 61,62 primary congenital 95,96 foreign bodygiant cells 52,61, 62 primary open-angle vii fornix-basedconjunctival flaps, pupil block 80 trabeculectomy 98 Scheie's procedure 94 foscarnet, CMV retinitis 70 secondary open-angle 42 fourth nerve palsy 8,51, 52 Sturge-Weber's syndrome 90 head posture 66 systemic drugs treatment 39,40 foveal avascular zone,sickle cell disease 68 trabeculotomy 94 foveal hypoplasia 58 treatment 39,40 frontal lobe lesion 22 glaucoma surgery, sympathetic ophthalmia Fuch's heterochromic cyclitis,cataract 44 surgery 74 glycerol,glaucoma 40 fungal keratitis 101-2 glycosylatedhaemoglobin assay 69,70 goblet cells GABA 2 dryeye 50 gancyclovir 20 stem cell deficiency 48 CMV retinitis 70 gold foreign bodies 62 gases, retina surgery 81,82 Goldenhai-'s syndrome 109,110 genetic disorders 12,93, 94, 97,98 limbal dermoids 76 Gerstmann's syndrome,dominant parietal Goldmann perimetry 56 lobe 120 gonioscopy 7, 8 giant cell arteritis 25,26 goniotomy,glaucoma 94 giant cells 52,61, 62 gonococcal conjunctivitis 22 Giemsa stain Gorlin's syndrome 94 fungal keratitis 102 grade C PVR, pneumatic retinopexy 14 herpes virus infections 122 Gram stain diagnoses 9, 10 glaucoma granular dystrophy 54 angle-closure 18,28, 81, 82 Grave's disease,pregnancy 114 aniridia 58 growth failure, de Morsier's syndrome 114 and cataract 78 Guillain Barre's syndrome 12 chronic primary angle-closure 81,82 gyrateapathy 58,112 Haab's striae 96 Horner's pupil 125,126 HAART see highly active anti-retroviral therapy Horner's syndrome 2, 51, 52,126 haemophilia A 118 cluster headaches 98 Haemophilus 74,106 heterochromic iridis 90 haemorrhage Hughe's procedure, basal cell carcinoma 112 expulsive suprachoroidal 13,14 Humphrey perimetry 56 IPCV 4 Hunter's syndrome 50,58 retinal 14 Hurler's syndrome 50,58 suprachoroidal 13, 14,18 Hutchinson's teeth 2 vitreous 16 HVA see homovanillic acid hard exudate, fovea 10 hydrops 50 HbSC, sickle cell disease 68 Down's syndrome 56 HbSThal, sickle cell disease 68 hydroxyamphetamine head posture 65,66 anisocoria 126 headaches miotic pupil 126 cluster headaches 98 hydroxylchloroqine pituitaryapoplexy 122 retinal pigment epithelial dysfunction 2 healon 84 vortex keratopathy 16 Henderson Patterson bodies 122 hyperemesis gravidarum 8 hepatitis B vaccine, uveitis 96 hypermetropia hereditary diseases accommodative esotropia 80 familial exudative vitreoretinopathy 22 chronic primary angle-closure glaucoma 82 Leber's hereditary optic neuropathy 9, 10 hypersegmentation, neutrophils, pale optic maculopathies 23,24 disc 46 retinal capillary haemangioma 20 hypertension 4, 24 retinoblastoma 12 see alsocorticosteroid-induced ocular Hermansky-Pudlak syndrome 42 hypertension herpes virus infections 121,122 hypertropia,divisional oculomotor nerve Herpes viruses, acute retinal necrosis 20 palsy 52 Herpetic Eye Disease Study 103,104 hyphaema 23,24 herpetic keratitis 74 hypopituitarism 122 acyclovir 104 hypopyon 44 herpetic stromal keratitis,topical steroids 104 hypotony Hess charts, muscle palsy 32 ciliary bodyablation 54 heterochromic iridis 89,90 peri bulbar anaesthesia 64 high scleral buckling 34 hypotropia highly active anti-retroviral therapy (HAART), Brown's syndrome 104 CMV retinitis 37,38, 44 divisional oculomotor nerve palsy 52 Hirschsprung's disease,heterochromic iridis 90 HIV-positive patients 20 ICG see indocyanine green homocystinuria, serum methionine 16 idiopathic polypoidalchoroidal vasculopathy homonymus hemianopia 74 (iPCV) 3,4 homonymus quadrantanopia 74 implant, lens see lens implant homovanillic acid (HVA), Riley-Day's impression cytology,superior limbic syndrome 100 keratoconjunctivitis 124 'in-the-bag' intraocular lens implantation 14 junctional scotoma 74 Nd:YAG laser capsulotomy 80 juvenile chronic arthritis 59, 60 indocyanine green (leG) 4, 83,84 juvenile idiopathic arthritis, lens implant 30 indomethacin, vortex keratopathy 16 juvenile xanthogranuloma, Touton giant cells 62 infantileesotropia 11,12 inferior oblique overaction 12 Kaposi's sarcoma 119,120 inferior rectus entrapment, blowout kappa angle, Rap 34 fracture 78 Kayser-Fleischer's ring 1, 2 interleukins, uveitis 96 Kearne-Sayre's syndrome, deafness 120 internuclear ophthalmoplegia 4, 52,63, 64 keratoconjunctivitis sicca, bone marrow intorsion,divisional oculomotor nerve transplantation 124 palsy 52 keratoconus 49,50 intraocular lens calculation, biometry 84, keratomalacia 50 85,86 keratometry, intraocular lens calculation 86 intraocular pressure (lOP) ketoconazole,cyclosporin 64 causes 18 Klippel-Trenaunay-Weber syndrome 38 ciliary bodyablation 54 decreased 52 lacrimal bone 28 expulsive suprachoroidal haemorrhage 14 lacrimal fossa,orbital lymphoma 76 glaucoma 42 lamellar cataracts 117,118 Horner's syndrome 52 Langhan's giant cells 62 measurement 55, 56 laser assisted in-situ keratomileusis (LASIK) 5, 6 normal tension glaucoma 56 laser iridotomy 27,28 raised 24,27, 28, 32 chronic primary angle-closure glaucoma 82 trabeculectomy 98 laser photocoagulation, central serous intravenous antibiotic 18 retinopathy 40 involutional entropion 87,88 laser treatment, branch retinal vein involutional ptosis 15,16 occlusion 36 ionising radiation 79,80 LASIK see laser assisted in-situ keratomileusis lOP see intraocular pressure latanoprost 8,89, 90 IPCV see idiopathic polypOidalchoroidal contraindications 54 vasculopathy macular oedema 2 ipsilateralinferior oblique myectomy 8 Sturge-Weber's syndrome 28 ipsilateraliris hyperpigmentation, Sturge-Weber's latent nystagmus 81,82 syndrome 90 lateral canthal laxity 88 ipsilateral medial longitudinal fasciculus 4 lattice degeneration 4, 31,32, 74 ipsilateralsuperior oblique tuck 8 lattice dystrophy 79,80 iridocorneal endothelial syndrome 78 Leber's congenital amaurosis 58,112 iridoschisis 99,100 deafness 120 iridotomy 4 Leber's hereditary optic neuropathy 9, 10 iris bombe 28 lefttrochlear nerve palsy 51, 52 iritis viii lens implant irrigation,alkaline burns 77,78 'in-the-bag'intraocular lens implantation 14 ischaemic optic neuropathy, YEP 42 juvenile idiopathic arthritis 30 isoniazide,optic neuropathy 114 piggyback intraocular 117,118 lens subluxation, Nd:YAG laser capsulotomy 80 marginal keratitis 99,100 leukocoria 43,44 mastering MCQs v-ix levator advancement, involutional ptosis 16 megalocornea 54 levator function 68 Meige's disease 48 levodopa 12 melanoma, choroidal 35,36 lid coloboma, Goldenhar's syndrome 110 membranous conjunctivitis, adenoviral lid retraction, strabismus surgery 32 keratoconjunctivitis 48 limbal-based conjunctival flaps, meningioma 111,112 trabeculectomy 98 menlanocytic hamartoma, limbal conjunctival excision, Mooren's ulcer 98 neurofibromatosis 60 limbal dermoids 75,76 mepacrine, vortex keratopathy 16 Goldenhar's syndrome 110 methanol, optic neuropathy 114 limbal incision, astigmatism 110 methenamine silver, fungal keratitis 102 limbal ischaemia, alkaline burns 78 methoxyflurane, crystallineretinopathy 2 limbal stem cell failure metipranolol, uveitis 96 aniridia 58 microaneurysms, sickle cell disease 68 ionising radiation 80 microangiopathy, radiation retinopathy 80 Lipschutz bodies,herpes virus infections 122 microphthalmos, Goldenhar's syndrome 110 Lisch's nodules, neurofibromatosis 60 microspherophakia 28 Louis-Bar syndrome 38 middle temporal gyrus lesion,akinetopsia 120 lower lid retractor laxity 88 Miller-Fisher's syndrome 11,12 lysinedeficiency, zonular dehiscence 18 miosis, perimetry 56 miotic pupil adrenaline 126 macrocytosis, neutrophils, pale optic disc 46 cocaine test 126 macular exudate, benign intracranial Horner's syndrome 126 hypertension 68 miotics, VEP 42 macular holes 39,40 mitomycin C 47,48 macular involvement, PORN 44 Mooren's ulcer 97,98 macular oedema Moraxella 10 cataract surgery 74 Morquio's syndrome 50 cystoid macular oedema 38 mucocele, congenital nasolacrimal duct diabetes mellitus 9, 10,34 obstruction 26 drugs causing 2 mucopolysaccharidoses 49,50, 58 par planitis 86 deafness 120 maculopathies 23,24 multiple basal cell carcinoma 93,94 malignant glaucoma 17, 18,28 multiple choroiditis, vitritis 84 malignant hypertension vi multiple evanescent white dot syndrome 115, mannitol, glaucoma 40 116 Mantoux test 60 multiple sclerosis 6 Marcus-Gunn jaw-winking syndrome, ptosis internuclear ophthalmoplegia 64 surgery 68 myelinated nerve fibres 34 Marfan's syndrome 69,70 Munson's sign, pellucid marginal echocardiogram 16 degeneration 104 zonular dehiscence 18 muscle atrophy,botulinum toxin 26 muscle palsy 31, 32 non-steroidal anti-inflammatorydrugs 90 Mustarde rotation flap,basal cell orbital myositis 88 carcinoma 112 normal tension glaucoma 55, 56 mutations Norrie's disease 98 Leber's hereditary optic neuropathy 9, 10 deafness 120 retinoblastoma 12 North Carolina macular dystrophy 24 myasthenia gravis 61, 62 nuclear sclerosis 78 ptosis surgery 68 nucleosides 38 mydriasis,divisional oculomotor nerve palsy 52 nucleus prolapse 14 mydriatic pupil 126 numerical figures, examiner's tricks vii myelinated nerve fibres 33, 34 nyctalopia(night blindness) 50, 58, 112 myopia nystagmus 52, 59, 60 corticosteroid-induced ocular hypertension de Morsier's syndrome 114 24 latent 81, 82 LASIK 6 local ising value 52 pigment dispersion syndrome 4 Stickler's syndrome 6 ocular albinism 98 zonular dehiscence 18 ocular bobbing, coma 116 myopic refraction 111, 112 ocular flutter, cerebellar lesion 116 myotonic dystrophy 16 ocular motilitydisorders 65, 66 ocular movements, abnormal 3, 4, 51, 52 naevus, choroidal 35, 36 ocular myoclonus, pontine stroke 116 naevus of Ota, heterochromic iridis 90 ocular oscillation,superior oblique nanophthalmos 89, 90 myokymia 114 nasolacrimal duct obstruction 27, 28, 74 oculoauriculovertebral dysplasiasee Goldenhar's Nd:YAG laser capsulotomy 79,80 syndrome Neisseria 10 oculocardiac reflex 29, 30 Neisseria gonorrhea 22, 76 oculomotor nerve, aberrant regeneration 3, 4 Neisseria meningitidis 76 Oil red 0 (stain),sebaceous cell carcinoma, neonatal conjunctivitis 21 eyelid 46 nephropathy 10 ONTT see Optic Neuritis Treatment Trial neuro-syphilis 118 opacification, piggyback intraocular lens neuroblastoma 52 implant 118 periorbital haematoma 118 ophthalmic arteryaneurysm 124 neurofibromatosis 38, 59, 60 ophthalmic arteryocclusion 69, 70 meningioma 112 ophthalmic herpes zoster 65, 66 neuropathy 10 ophthalmoplegia 4, 8, 63, 64, 66 nicotinic acid, macular oedema 2 pituitaryapoplexy 122 night blindness (nyctalopia)50, 58, 112 opsoclonus 52, 116 NNRTI see non-nucleoside reverse transcriptase optic atrophy 45, 46, 58, 118 inhibitors optic nerve trauma 96 non-nucleoside reverse transcriptase inhibitors optic canal enlargement 123, 124 (NNRTI)38 optic disc cupping 96 non-proliferativediabetic retinopathy 112 optic disc swelling 109, 110 non-specific urethritis, and conjunctivitis 106 optic nerve glioma 124 optic nerve hypoplasia,de Morsier's periorbital haematoma 117,118 syndrome 114 peripapillarysubretinal neovascular membrane optic nerve meningioma 124 formation, disc drusen 108 normal tension glaucoma 56 perivascular pigmentary changes 6 optic nerve trauma 95,96 Peter's anomaly,cloudy cornea 54 optic neuritis 6, 10,20 phacoemulsification 17,18 Optic Neuritis Treatment Trial (ONTT) 6 phacolytic glaucoma 28 optic neuropathy, drugs causing 114 phacomorphic glaucoma 28 optokinetic response 21,22 phaeochromocytoma, retinal capillary congenital nystagmus 60 haemangioma 20 oral contraceptives 102 phakic/pseudophakic patients uveitis 96 argon laser trabeculoplasty 28 oral corticosteroids 6 pneumatic retinopexy 14 orbicularis oculi, stripping 48 phakomatoses 37,38 orbital cellulitis 32,64 phenothiazine, retinal pigment epithelial orbital emphysema 31,32 dysfunction 2 orbital lymphoma 75,76 phenylephrine 2 orbital myositis 87,88 phopholine iodide,AC!A ratio 96 orbital rhabdomyosarcoma 63,64 photocoagulation ornithine level 58 cerebellar haemangioblastoma 20 oscillation,ocular 114 familial exudative vitreoretinopathy 22 phaeochromocytoma 20 P-ANCA see perinuclear ANCA photodynamic therapy (PDT) 71,72 painless visual loss 10 photoreactive keratectomy (PRK) 5, 6 pale optic disc 45,46 phrasing, examiner's tricks v-vi pan-photocoagulation 118 phthisis bulbi 30 Pancoast's tumour 126 phthisis, Ciliarybody ablation 54 pan uveitis 20 PI see protease inhibitors papilloedema 109,110 piggyback intraocular lens implant 117,118 par planitis 85,86 pigment abnormalities, sympathetic Parinaud's syndrome 22 ophthalmia 20 convergence retraction nystagmus 118 pigment deposition parkinsonism 12 angles 7, 8 patent peripheral gonioscopy,chronic primary Sampaolesi's line 8 angle-closure glaucoma 82 pigment dispersion syndrome 3, 4 pavingstone retinal degeneration 107,108 pigmentary changes PDT see photodynamic therapy bone-spicule 6 pellucid marginal degeneration 103,104 perivascular 6 pendular nystagmus 52 pigmentary glaucoma 4 peri bulbar anaesthesia 63,64 pilocarpine 2,28 oculocardiac reflex 30 anisocoria 126 perifoveal capillary permeability,CMO 106 contraindications 54 perimetry 55, 56 mydriatic pupil 126 perinuclear ANCA (P-ANCA) 10 pinealoma 52 periodic acid schiff (PAS), fungal keratitis 102 pituitaryapoplexy 121,122 plasma lipoprotein B 58 propriobacterium acne, Nd:YAG laser platinum foreign bodies 62 capsulotomy 80 pleomorphic adenoma 101,102 proptosis 71,72 plus disease 107,108 protease inhibitors (PI) 38 pneumatic retinopexy 13,14 pseudo Argyll-Robertson pupil 4 poliosis 19,20 pseudo von Graefe's sign 4 polyarteritisnodosa 10 pseudoexfoliation syndrome 41,42 polychromatic cataract 16 zonular dehiscence 18 polycythaemia,cerebellar Pseudomonas aeruginosa 10 haemangioblastoma 20 pseudophakic patients,latanoprost 90 pontine stroke, ocular myoclonus 116 pterygium recurrence, mitomycin C 48 PORN see progressive outer retinal necrosis ptosis port-wine stain,Sturge-Weber's syndrome 90 cluster headaches 98 posterior capsule rupture 18 divisional oculomotor nerve palsy 52 cataract surgery 102 Horner's syndrome 52 posterior capsule thickening 14,65, 66 myasthenia gravis 62 posterior polar cataract 101,102 surgery 67,68 posterior polymorphous dystrophy 78 pulsatile exophthalmos, neurofibromatosis 60 posterior subcapsular cataract punctate keratitis,epiblepharon 91,92 intravitreal gas use 82 punctate keratopathy, aqueous tear neurofibromatosis 60 deficiency 102 par planitis 86 pupil block glaucoma, Nd:YAG laser posterior synechiae, par planitis 86 capsulotomy 80 posterior vitreous detachment 40,86 pupil, fixed dilated 1, 2 pregnancy 8,113, 114 pupillary block 28 pressure, intraocular see intraocular pressure pupillary rigidity,plus disease 108 primary angle-closure glaucoma 81,82 PVR see proliferative vitreoretinopathy primary biliarycirrhosis 2 primary congenital glaucoma 95, 96 quinidine, uveitis 96 primary open-angle glaucoma vii PRK see photoreactive keratectomy progressive astigmatic myopia,keratoconus 50 racoon's sign 117,118 progressive outer retinal necrosis (PORN) radial perineuritis, acanthoamoeba keratitis 46 43,44 radiation,ionising 79,80 progressive supranuclear palsy 11,12 radiation retinopathy,microangiopathy 80 proliferative diabetic retinopathy 23, 24,53, 54 radiation,sebaceous cell carcinoma, eyelid 46 cataract surgery 74 radiotherapy,orbital lymphoma 76 proliferative retinopathy 68 Ramsay Hunt's syndrome, unilateral facial nerve proliferativevitreoretinopathy (PVR) 15,16,115, palsy62 116 Raynaud's phenomenon 126 pneumatic retinopexy 14 reading, superior oblique myokymia 114 sickle cell disease 68 recurrent corneal erosion syndrome, lattice prophylactic antibiotic 32 dystrophy80 prophylactic systemic steroid treatment, Refsum's disease 58 uveitis 30 Reiger's syndrome 2 relative afferentpupillary defect Scheie's syndrome 50 arteryocclusion 70 Schwalbe's line 8 proptosis 72 Schwann cell tumours, acoustic neuroma 76 renal failure 24 scleral buckle retina surgery gases 81,82 PVR 116 retinal capillary haemangioma 19,20 retinal detachment 103,104 retinal detachment 6, 14, 18,22 scleral spur 8 acute retinal necrosis 20 sclerocornea, cloudy cornea 54 lattice degeneration 32 sclerosed vessels, lattice degeneration 32 photodynamic therapy 72 scotoma, dense paracentral, normal tension retinal dialysis 33,34 glaucoma 56 rhegmatogenous 99,100, 121, 122 screening, ROP 34 scleral buckle 103,104 sebaceous cell carcinoma, eyelid 45,46 trauma 34 secondary open-angle glaucoma 42 retinal dialysis,retinal detachment 33,34 see-saw nystagmus 52 retinal haemorrhages 14 Serratia marcescens 10 retinal ischaemia 24 serum ACE, sarcoidosis 58,60 retinal periphlebitis,sarcoidosis 60 serum folate 46 retinal pigment epithelial dysfunction,drugs serum FT A Abs, pale optic disc 46 causing 2 serum methionine, homocystinuria 16 retinal thickening 10 serum phytanic acid 58 retinitis pigmentosa 98,119, 120 shaken babysyndrome 13,14 disc drusen 108 shallowing, anterior chamber 17,18 retinoblastoma vi, 11, 12,43, 44 sickle cell anaemia 24,118 retinopathy of prematurity (ROP) 22,33, 34 glycosylatedhaemoglobin assay 70 retinoschisis 68,98, 108 sickle cell disease 67,68 rhegmatogenous retinal detachment 99,100, siderosis,b-wave amplitude reduction 40 121,122 silicone oil 83,84 rhinorrhoea, cluster headaches 98 intraocular lens calculation 84,86 rifabutin,uveitis 96 silver foreign bodies 62 right abducence nerve palsy,botulinum toxin 26 silver nitrate,superior limbic right superior oblique palsy 7, 8 keratoconjunctivitis 124 Riley-Day'ssyndrome 99,100 sixth nerve palsy,head posture 66 ROP see retinopathy of prematurity skew deviation 4 Roth spots, candida endophthalmitis 106 skin anergy 60 rubella, congenital 77,78 skip lesion,giant cell arteritis 26 deafness 120 SLE see systemic lupus erythematosus sleep, essential blepharospasm 48 Sampaolesi's line 8 Sorby's macular dystrophy 24 Sanfilippo's syndrome 50 spasmus nutan 116 sarcoidosis 59, 60 spastic entropion 88 Langhan's giant cells 62 sphenoidal meningioma, optic canal enlargement serum ACE 58 124 Scheie's procedure, glaucoma 94 Staphylococcusaureus, marginal keratitis 100 Scheie's stripe 8 Staphylococcusepidermidis 18 Stargardt's macular dystrophy 24 suprachoroidal haemorrhage 18 stem cell deficiency 47, 48 symblepharon, Steven-johnson'S syndrome 50 STER MRI sequence, thyroid eye disease 26 sympathetic ophthalmia 43, 44 steroids ciliary body ablation 54 adenoviral keratoconjunctivitis 48 pigment abnormalities 20 central serous retinopathy 40 synechiae 28, 30 see a/50corticosteroids syphilis steroids, systemic congenital 2, 120 corticosteroid-induced ocular deafness 120 hypertension 24 ectopia lentis 16 orbital myositis 88 neuro-syphilis 118 Tolosa-Hunt's syndrome 92 zonular dehiscence 18 uveitis 30 syringomyelia 126 steroids, topical systemic corticosteroids see corticosteroids, alkaline burns 78 systemic corticosteroid-induced ocular hypertenSion systemic drugs treatment, glaucoma 39, 40 24 systemic lupus erythematosus (SLE) 91, 92 herpetic stromal keratitis 104 systemic steroids see steroids, systemic Steven-johnson'S syndrome 49 acetazolamide 54 tamoxifen Stickler's syndrome 5 crystalline retinopathy 2 strabismus surgery 31, 32 vortex keratopathy 16 Streptococcus 74 Tay-Sach's disease 58 streptokinase, uveitis 96 temporal artery, giant cell arteritis 26 streptomycin, optic neuropathy 114 temporal clear corneal incision 109, 110 stroke, carotid artery stenosis 72 tendon reflexes, myasthenia gravis 62 STUMPED mnemonic, cloudy cornea 54 Tenon's cyst 29, 30 Sturge-Weber's syndrome 27, 28, 38, 89, 90 Tensilon test, myasthenia gravis 62 subconjunctival antibiotics, cataract surgery 88 Tenzel flap, basal cell carcinoma 112 subluxated lens 15, 16 Thayer-Martin medium 22 Sudan black (stain), sebaceous cell carcinoma, third nerve palsy 52 eyelid 46 oculomotor nerve aberrant regeneration 4 sulphate oxidase deficiency, zonular pilocarpine 2 dehiscence 18 thyroid eye disease 25, 26 sulphite oxidase deficiency, urine thyroidectomy 126 thiosulphate 16 TIA see transient ischaemic attacks superficial punctate keratitis, SLE 92 timolol superior limbic keratoconjunctivitis 123, 124 contraindications 54 superior oblique myokymia 113, 114 Sturge-Weber's syndrome 28 superior rectus palsy, head posture 66 Tolosa-Hunt's syndrome 91, 92 superior rectus resection 32 topical corticosteroids, stromal keratitis 104 superior sulcus deformity 91, 92 topical steroids see steroids, topical superior visual field defect, dermatochalasis 91, Touton giant cells 92 Erdheim-Chester's disease 62 juvenile xanthogranuloma 62 V pattern, Brown's syndrome 104 xanthoma disseminatum 62 valve of Hasner, congenital nasolacrimal duct oxocariasis 43,44 obstruction 26 oxoplasmosis 85,86 varicella zoster rabecular meshwork 8,28, 78 acute retinal necrosis 20 pseudoexfoliation syndrome 42 PORN 44 rabeculectomy 94 vein, enlarged superior ophthalmic 21,22 24-hours post 81,82 VEP see visual evoked potential conjunctival flaps 97,98 vermiform eyemovement 118 endophthalmitis 73,74 vernal keratoconjunctivitis 109,110 lOP 18 verteporfin 72 rabeculodialysis 94 vertical diplopia,fourth cranial nerve palsy 52 rabeculotomy, glaucoma 94 videokeratography, intraocular lens ransient ischaemic attacks (TIA),carotid artery calculation 86 stenosis 72 vigabatrin 1, 2 rauma, optic nerve 95, 96 b-wave amplitude reduction 40 rauma, retinal detachment 34 virus infections,herpes 121,122 raumatic angle recession 78 viscoelastic substances 83,84 ricks, examiners' v-ix visual evoked potential (VEP) 41,42 risomy 21,Down's syndrome 56 visual field defects 2, 73,74 uberculosis, Langhan's giant cells 62 visual loss 69,70 uberous sclerosis 38 pituitaryapoplexy 122 ypeII collagen, PRK 6 vitamin A deficiency 49,50 lzanck smear, herpes virus infections 122 vitamin Bl deficiency,Wernicke's encephalopathy 8 Jhtoff's phenomenon 10 vitamin B12 deficiency 46,123, 124 JKPDS see United Kingdom Prospective Diabetes vitrectomy surgery Study gases 82 Jlcers, dendritic 65,66 light-perception vision 18 Jnilateral blepharitis, sebaceous cell carcinoma, proliferative diabetic retinopathy 54 eyelid 46 PVR 16 Jnilateral facial nerve palsy 61,62 silicone oil 84 Jnited Kingdom Prospective Diabetes Study vitreoretinopathy see familial exudative (UKPDS) 10,105, 106 vitreoretinopathy;proliferative Jrine thiosulphate vitreoretinopathy ectopia lentis 16 vitreous haemorrhages, PVR 16,116 sulphite oxidase deficiency 16 vitreous haze, plus disease 108 Jsher's syndrome, deafness 120 vitritis 38,83, 84 JVeal melanoma, pregnancy 114 Vogt-Koyanagi-Harada's syndrome (VKH) 20 JVeitis 19,20, 95, 96 Dalen-Fuchs' nodules 44 acyclovir 104 deafness 120 cataract surgery 29,30 von Hippel-Lindau's syndrome 38 cyclosporin 64 retinal capillary haemangioma 20 juvenile chronic arthritis 60 vortex keratopathy, drugs causing 16 Waardenburg's syndrome, heterochromic Wilm's tumour, aniridia 58 iridis 90 Wilson's disease 2 wall-eyedbilateral internuclear ophthalmoplegia Wright's stain,herpes virus infections 122 (WEBINO) 4 Wyburn-Mason syndrome 38 Watzke-Allen's sign 40 WEBINO see wall-eyedbilateral internuclear X-linked congenital stationarynight ophthalmoplegia blindness 58,112 wedge resection, pellucid marginal xanthoma disseminatum, T outon giant cells 62 degeneration 104 xeroderma pigmentosa 94 Wegener's granulomatosis 10 xerophthalmia 50 Wei I-Marchesani syndrome,zonular dehiscence 18 zonular cataracts 117,118 Wernicke's encephalopathy 7, 8 zonular dehiscence 17, 18,42 West's syndrome 2